Sunteți pe pagina 1din 136

TEMA No 1

INTRODUCCION A LAS ECUACIONES


DIFERENCIALES

De…nición.
Una ecuación diferencial, es una ecuación que establece una relación de una
o más varibales dependientes y sus derivadas con respecto a una o más variables
independientes.

Clasi…cación de las Ecuaciones Diferenciales.


Las ecuaciones diferenciales se clasi…can de acuerdo, con el tipo, el orden y
la linealidad.

Clasi…cación Según el Tipo.

a). Ecuaciones diferenciales ordinarias (E.D.O.).


Una ecuación diferencial ordinaria, es una ecuación que solo contiene derivadas
ordinarias de una o más variables dependientes con respecto a una sola variable
independiente.

Una ecuación diferencial ordinaria, de variable independiente x y una sola


varibale dependiente y, simbolicamente podemos escribir

dy d2 y dn y
F x; y; ; 2 ; :::; n =0
dx dx dx

ó bien

F x; y; y 0 ; y 00 ; y 000 ; :::; y (n) = 0,

donde F es una función a especi…car.

Ejemplos.
dy
a). = 3x + 1 E.D.O. de varibale independiente x y dependiente y.
dx

b). y 00 + xy 0 = 3 E.D.O. de variable independiente x y dependiente y.

d2 y
c). at = 0 E.D.O. de variable independiente t y dependiente y.
dt2

1
d). y {v y 0 = C E.D.O. de variable independiente x y dependiente y,
donde C es una constante.

e). x2 y 2 dx xydy = 0 ,
xydy = x2 y 2 dx ,

dy x2 y 2
= .
dx xy

E.D.O. de variable independiente x y dependiente y.


b). Ecuaciones Diferenciales Parciales (E.D.P.).
Una ecuación diferencial parcial, es una ecuación que contiene derivadas
parciales, de una o más variables dependientes respecto de dos o más variables
independientes.

Ejemplos.

@u @u
a). =0.
@x @y
E.D.P. , variables independientes x, y; variable dependiente u.

@u @v
b). x +y =C
@x @y
E.D.P. , variables independientes x, y; variable dependiente u, v y C es una
Ctte.

@2u 2
2 @ v
c). + t =t
@t2 @w2
E.D.P. , variables independientes t, w; variable dependiente u, v.

@2w @2w @2w


d). + + =0.
@x2 @y 2 @z 2
E.D.P. , variables independientes x, y ,y z; variable dependiente w. Es la
ecuación de Laplace.

Clasi…cación Según el Orden.


El orden de una ecuación diferencial, es el orden de la derivada más alta,
que aparece o que se encuentra en la ecuación.
Una ecuación diferencial ordinaria de orden n, de variable independiente x
y variable dependiente y, se escribe:

2
F x; y; y 0 ; y 00 ; y 000 ; :::; y (n) = 0.

Ejemplos.

a). xdy + ydy = 0 . E.D.O. de primer orden.

d2 y dy
b). +x + x2 y = 0 . E.D.O. de segundo orden.
dx2 dx
2
c). (y 0 ) + xy 00 + y = 0 . E.D.O. de segundo orden.
@2u @2y
d). =t. E.D.P. de segundo orden.
@t2 @w2
Clasi…cación Según la Linealidad y No Linealidad.

a). Ecuación Diferencial Lineal.

Una E.D.O. lineal, de variable dependiente y y variable independiente x,


puede escribirse como sigue:
a0 (x) y (n) + a1 (x) y (n 1) + + an 1 (x) y 0 + an (x) y =
g (x) (1)

Donde g (x) y los coe…cientes a0 (x) ; a1 (x); :::; an 1 (x); an (x) dependen
solamente de la variable x y no de y.

Ejemplos.

dy
a). = 3x E.D.O. lineal de primer orden.
dx

d2 y
b). + sen(t + 1)y = C E.D.O. lineal de segundo orden.
dt2
c). y 00 + y 0 + ky = 0 E.D.O. lineal de segundo orden, donde k es una
constante.

d). y {v y = e x . E.D.O. lineal de cuarto orden no lineal.


Ecuación Diferencial No Lineal.
Una ecuación diferencial ordinaria que no es de la forma (1), se llama
ecuación diferencial ordinaria no líneal.

Ejemplos.

a). y 0 = ey . E.D.O. no lineal de primer orden.

3
d3 y
b). + cos(y + 1)y = 0 E.D.O. de tercer orden no lineal.
dx3

c). (y 0 )2 + x = 0 E.D.O. de primer orden no lineal.

d). y 000 y 0 = cos(x + y) E.D.O. de tercer orden.

Ejemplos.
Clasi…car cada una de las siguientes ecuaciones diferenciales de acuerdo al
tipo, orden y linealidad.

d3 t
a). + y2 = C E.D.O. de tercer orden, no líneal.
dy 3

d2 y
b). (y 00 )2 + = x2 + sen x E.D.O. de segundo orden, no lineal.
dx2

@2u @2v
c). 2
+ =t E.D.P. de segundo orden.
@t @w2

d2 y
d). ex + (ln x)y = 0 E.D.O. de segundo orden, lineal.
dx2

e). y iv + ky = ln (x + y) E.D.O. de cuarto orden, no lineal, k, es una


constante.

Solución de Una Ecuación Diferencial.


Una solución de una ecuación diferencial es cualquier función f , de…nida en
un intervalo I, tal que, sí sustituida en dicha ecuación la reduce a una identidad
en el intervalo especi…cado. El intervalo I puede representar: un intervalo
abierto (a; b), cerrado [a; b], o in…nito (0; 1), etc.

Ejemplos.
1. La función y = xex es una solución de la ecuación diferencial lineal de
segundo orden

y 00 + 2y 0 + y = 0,

de…nida en el intervalo ( 1; 1).


Solución.
Las derivadas de la función y = xex son:

y 0 = ex + xex

y 00 = ex + ex + xex = 2ex + xex

4
Sustituyendo estas expresiones en la ecuación dada, tenemos

y 00 2y 0 + y = 2ex + xex 2 (ex + xex ) + xex


= 2ex + xex 2ex 2xex + xex

=
0:

sen x
2. La función y = es una solución de la ecuación lineal de primer
x
orden

xy 0 + y = cos x,

de…nida en el intervalo de (0; 1) :


Solución.
sen x
La derivada de la función y = es:
x
x cos x sen x
y0 = .
x2

Sustituyendo, en la ecuación inicial, tenemos.

x cos x sen x sen x


xy 0 + y = x +
x2 x

x cos x sen x sen x


= +
x x
x cos x sen x sen x
= +
x x x
= cos x:

3
3. Veri…car que la función y = 2 2 x2 , es una solución de la ecuación defer-
encial no lineal,
1
y0 4 (xy) 3 = 0,

de…nida en el intervalo (0; 1).


Solución.
3
La derivada de la función y = 2 2 x2 es:

5
3 5
y 0 = 2 2 2x = 2 2 x

Sustituyendo y 0 en la ecuación inicial, tenemos

1 5
h 3
i 31
y0 4 (xy) 3 = 2 2 x 4 x 2 2 x2

5 1
= 22 x 4 22 x = 0

5 1
= 22 x 22 2 2 x

= 0:

x4
4. Veri…car que la función y = es una solución de la ecuación no lineal.
16
1
y 0 = xy 2 ,

de…nida en el intervalo ( 1; 1).

Solución.
x4
La derivada de la función y = es:
16
x3
y0 = .
4

Reemplazando la derivada de la función en la encuación dada, tenemos


1
1 x3 x4 2

y0 xy 2 = x
4 16
1
x3 x4 2

= x
4 42

x3 x2
= x
4 4

x3 x3
=
4 4
= 0:

6
Soluciones Explicitas e Implicitas.
Una ecuación diferencial puede tener como soluciones explicítas e implicítas.
Soluciones Explícitas.
Se llama solución explicita de una ecuación diferencial a una función que se
puede escribir en la forma y = f (x), en el intervalo I.

Ejemplo.
sen x 3
Las funciones de los ejemplos anteriores y = xex , y = , y = 2 2 x2 son
x
soluciones explícitas de sus correspondientes ecuaciones diferenciales.
Soluciones Implícitas.
Se llama solución implícita de una ecuación diferencial a la función G(x; y),
si de…ne una o más soluciones explícitas en el intervalo I.

Ejemplo.
Veri…car que la función x2 + y 2 = 16 es una solución implicita de la ecuación
x
diferencial de primer orden y 0 = en el intervalo ( 4; 4) :
y
Solución.

La función x2 + y 2 = 16, derivamos implicitamente.

2x + 2yy 0 = 16.

Despejando y 0 tenemos
x
y0 = ,
y

que es idéntica a la ecuación inicial. Por otra parte, la función x2 + y 2 = 16


de…ne dos funciones implicítas
p p
y = 16 x2 y y = 16 x2 .

En consecuencia, la función x2 + y 2 = 16 es una solución implícita de la


x
ecuación y 0 = .
y
Ejemplo.
Muestre que xy +exy = 0 es una solución implicita de la ecuación diferencial

(x + xexy ) y 0 + y + yexy = 0.

Solución.

Derivamos implicitamente la función xy + exy = 0, tenemos

7
y + xy 0 + yexy + xy 0 exy = 0.

Luego

(x + xexy ) y 0 + y + yexy = 0.

Se entiende también por solución implícita, cuando la solución no se puede


expresar en la forma y = f (x).

Familia de Soluciones.
Una ecuación diferencial tiene generalmente un número in…nito de soluciones
es decir una familia de soluciones con un parámetro llamado constante arbitraria
C.

Ejemplos.
1. La ecuación diferencial de primer orden
y
y 0 = 2 , x 6= 0 ,
x
tiene por solución la familia de parabolas, y = Cx2 , donde C, es una con-
stante.

Solución.
Primeramente veri…quemos que la función y = Cx2 es una solución de la
ecuación planteada. La primera derivada es y 0 = 2Cx. Sustituyendo en la
ecuación, resulta

2y 2Cx2
y0 = 2Cx = 2Cx 2Cx = 0:
x x

Dando a C, distintos valores generamos la familia de parábolas.

C
2. La función y = + 1, donde C es una constante, es una solución de la
x
ecuación lineal de primer orden

xy 0 + y = 1,

en el intervalo (0; 1).


Solución.
C
Donde y 0 = Cx 2
= , sustituyendo en la ecuación inicial, veri…camos
x2

8
Parabolas

1 :pdf

Hiperbolas

2 :pdf

9
C C Cx C C C
xy 0 + y = x + +1= + +1= + + 1 = 1.
x2 x x2 x x x

Dando a C distintos valores generamos la falimia de soluciones.

3. Las funciones y1 = C1 sen 2x y y2 = C2 cos 2x, donde C1 y C2 son dos


constantes, son soluciones de la ecuación diferencial de segundo orden y 00 + 4y =
0:

Solución.
Las derivadas de la función y1 = C1 sen 2x son:

y10 = 2C1 cos 2x

y100 = 4C1 sen 2x

Vemos que:

y 00 + 4y = 4C1 sen 2x + 4C1 sen 2x = 0.

Por otra parte, las derivadas de la función y2 = C2 cos 2x son:

y20 = 2C2 sen 2x

y200 = 4C2 cos 2x

Así que:

y 00 + 4y = 4C1 cos 2x + 4C1 cos 2x = 0.

La suma de estas soluciones, y = C1 sen 2x + C2 cos 2x, también es una


solución de la ecuación dada. La prueba de esta solución es en forma análoga.

La grá…ca de las funciones y1 = C1 sen 2x, y2 = C2 cos 2x. y y = C1 sen 2x +


C2 cos 2x para C1 = C2 = 1, se muestran en las …guras de abajo.

10
Sen 2 x cos 2 x
2 2

1 1

2 1 1 2 3 4 2 1 1 2 3 4

1 1

2 2

cos 2 x Sen 2 x
2

2 1 1 2 3 4

TEMA No 2

ECUACIONES DIFERENCIALES DE PRIMER


ORDEN

En general una ecuación diferencial de primer orden se puede escribir de la


siguiente manera:

F (x; y; y 0 ) = 0 (Forma Implicíta)

Sí en está ecuación es posible despejar y 0 , se tiene:


dy
= f (x; y), o bien y 0 = f (x; y) (Forma Explícita)
dx

representa una ecuación diferencial de primer orden, donde f es una función


de dos variables x y y.
Problema de Valor Inicial.
El problema de valor inicial de una ecuación diferencial de primer orden

11
y 0 = f (x; y)

signi…ca, encontrar una solución de la ecuación diferencial en un intervalo I


que satisfaga la condición.
y (x0 ) = y0 ,

donde x0 es un número en el intervalo I, y y0 es un número real.


La relación y (x0 ) = y0 , se llama "condición inicial" o "valor inicial" de la
ecuación dada. Gra…camente signi…ca:

Soluciones de la ecuación diferencial.


Ejemplo.
Consideremos nuevamente la ecuación
2y
y0 =
x
y la condición inicial y (1) = 1.
Solución.
Representa un problema de valor inicial, sabemos que la ecuación tiene una
familia de soluciones y = Cx2 .
Para obtener una solución particular, que satisfaga la condición y(1) = 1, es
necesario sustituir x = 1 y y = 1 en la familia de soluciones: y = Cx2 .
Sí: x = 1 y y = 1 =) y = Cx2
1 = C(1)2 =) C = 1.
2
De modo que: y = x , es una solución particular de la ecuación dada, que
es una solución única. Geométricamente, esto es

15
y
10

-4 -2 0 2 4
x

12
Teorema de Existencia y Unicidad de una Solución (T.E.U.)
Sí en el problema de valor inicial y 0 = f (x; y) y y(x0 ) = y0 , la función
@f
f (x; y) y su derivada parcial respecto de y son continuas en una región
@y

rectangular R, en el plano xy y que contiene el punto (x0 ; y0 ), entonces existe


una y sóla una solución y = f (x) de la ecuación diferencial dada que satisface
la condición y = y0 cuando x = x0 .

Ejemplo.
Aplicar el T.E.U. para determinar si existe una solución única a el problema
de valor inicial.

y 0 = x3 + y 2 , y(1) = 2.

Solución.
@f
Vemos que f (x; y) = x3 + y 2 y = 2y, son continuas en cualquier región
@y
rectangular que contenga al punto (1; 2). Por tanto el T.E.U. garantiza una
solución única al problema de valor inicial en una región R que pasa por el
punto (1; 2).

Ejemplo.
Dado el problema de valor inicial.
3 2
y0 = y3 y y(2) = 0,
2

veri…car si se cumple el T.E.U.


Solución.
3 2
Donde f (x; y) = y 3 , su derivada parcial respecto de y es:
2
@f 23 1 1
= y 3 = p .
@y 32 3 y

13
@f
Desafortunadamente la no es continua puesto que no está de…nida para
@y
el punto y = 0, esto signi…ca que no existe una región rectangular R que contiene
al punto (2; 0). Su restricción es y > 0.

Por tanto T.E.U. no garantiza una solución única al problema dado, pues
no existe una región R por el cúal pase el punto (2; 0).

El problema de valor inicial, tiene una solución y = (x 2)3 , también y = 0


es una solución, una tercera solución también lo es

sí: x > 2
3
(x 2)
y=
0 sí: x < 2

y 25
20
15
10
5
0
1 2 3 4 5
-5 x

Ejemplo.
1
Dado el problema de valor inicial y 0 = xy 2 , y(0) = 0. Veri…car si se cumple
el T.E.U.
Solución.
1
Donde: f (x; y) = xy 2 , su derivada parcial para está función, resulta:

@f 1 1 x
= xy 2 = p .
@y 2 2 y

@f
Lamentablemente no es continua, tiene restricciones, puesto que no está
@y

de…nida en el punto y = 0, esto establece que no existe una región rectángular


R, que contenga al punto (0; 0). El T.E.U. no garantiza una solución única a el
problema de valor inicial. En la grá…ca se puede observar, que tiene al menos
x4
dos soluciones cuyas grá…cas pasan por (0; 0), siendo y = 0 y y = .
16

Pero, si el semiplano y > 0, existe una solución única en cualquier región

rectángular R. Así, por ejemplo para cualquier punto (x0 ; y0 ), y > 0. Para

14
(2; 1), existe una solución única, esto signi…ca que existe una región rectangular
R, que contiene al punto (0; 0).

ECUACIONES SEPARABLES
Si la ecuación diferencial de primer orden.
dy
= f (x; y),
dx
se puede escribir en la forma diferencial.

M (x) dx + N (y) dy = 0

o bien
dy M (x)
= .
dx N (y)

se llama ecuación separable o ecuaciones de variables separables.


Método Solución.
La ecuación diferencial separable.

dy M (x)
= (1)
dx N (y)

también se puede escribir


dy
N (y) = M (x),
dx
dy
donde = y 0 = f 0 (x).
dx
Supongase que y = f (x) es la solución de (1), entonces

N (f (x))f 0 (x) = M (x).

Integrando respecto de x, tenemos.


R R
N (f (x))f 0 (x) dx = M (x) dx + C.

Como dy = f 0 (x) dx y y = f (x).

15
R R
N (y) dy = M (x) dx + C

ó bien
R R
M (x) dx + N (y) dy = C.

En el caso particular sí N (y) = 1, en la ecuación (1) se reduce


dy
= M (x)
dx
ó bien

dy = M (x) dx

Integrando
R R
dy = M (x) dx + C.

Por tanto,
R
y= M (x)dx + C

obtenemos una solución explicita.

Ejemplo.
Resolver la ecuación diferencial
dy y 1
= .
dx x+3
Solución.
La ecuación es de variables separables, pues
dy dx
= .
y 1 x+3
Integrando
Z Z
1 1
dy = dx + C1
y 1 x+3
ln jy 1j = ln jx + 3j + ln jC1 j,

donde C1 = ln jCj, luego aplicando la propiedad de logaritmos, tenemos

ln jy 1j = ln jC (x + 3)j
y 1 = C(x + 3).

Obtenemos una

y = C(x + 3) + 1

16
solución explicita.
Ejemplo.
Resolover la ecuación
dy y 2 (x 1)
= 2 3 .
dx x (2y y)

Solución.
Separando variables, tenemos

dy y2 x 1
= 3
dx 2y y x2

2y 3 y x 1
dy = dx.
y2 x2
Integrando, tenemos:
Z Z
2y 3 y x 1
dy = dx + C
y2 x2
Z Z Z Z
2y 3 y x 1
dy dy = dx dx + C
y2 y2 x2 x2
Z Z Z Z
1 1 1
2ydy dy = dx dx + C
y x x2
Z Z Z Z
1 1 2
2 ydy dy = dx x dx + C
y x
y2 x 1
2 ln jyj = ln jxj +C
2 1
1
y2 ln jyj = ln jxj + +C
x
1
y2 ln jxj ln jyj =C
x
1
y2 (ln jxj + ln jyj) = C.
x
Obtenemos una
1
y2 ln jxyj = C
x
solución implicita.

Ejemplo.
Resolver la ecuación.

17
p p
x 1 + y 2 dx + y 1 + x2 dy = 0.

Solución.
Separando variables se tiene.
p p
y 1 + x2 dy = x 1 + y 2 dx
y x
p dy = p dx.
1 + y2 1 + x2

Integrando
Z Z
y x
p dy=- p dx + C.
1+y 2 1 + x2

Aplicando un cambio de variable a ambos integrales, tenemos.


Si: u = 1 + y 2 ) du = 2ydy y Sí: z = 1 + x2 ) dz = 2xdx
du dz
= ydy. = xdx .
2 2
Z Z
1 du 1 dz
p = p +C
2 u 2 z

Z Z
1 1 1 1
1 du = 1 dz + C
2 u2 2 z2
Z Z
1 1 1 1
u 2 du = z 2 dz + C
2 2
1 1
1 u2 1 z2
= +C
2 12 2 12
1 1
u2 = z 2 + C.

Sí: u = 1 + y 2 y z = 1 + x2 , tenemos
1 1
(1 + y 2 ) 2 = 1 + x2 2
+C
1 1
1 + x2 2
+ 1 + y2 2
= C.

Así, obtenemos una solución implícita


p p
1 + x2 + 1 + y 2 = C.

18
Ejemplo.
Resolver la ecuación.

y 0 sen x = y ln y,

sujeto a la condición inicial y = e.


2
Solución.
Se trata de un problema de valor inicial, separando variables, tendremos
dy
sen x = y ln y
dx
dy sen x = y ln ydx

dy dx
= .
y ln y sen x

Integrando
Z Z
1 1
dy = dx + C1 .
y ln y sen x

1
Sí: u = ln y ) du = dy:
y
Z Z
du 1
= dx + C1
u sen x
x
ln juj = ln tan + ln jCj.
2

Donde C1 = ln jCj,
x
ln jln yj = ln C tan
2
x
ln jyj = C tan .
2

Por tanto, la solución general es:


x
C tan
y=e 2.

Sustituyendo la condición inicial x = y y = e, resulta.


2

19
2
C tan
e=e 2

e = eC .

Aplicando logaritmo neperiano, tenemos

ln e = C ln e ó e1 = eC

1=C C = 1.

Por tanto, la solución pariticular es


x
tan
y=e 2.

Ejemplo.
Resolver la ecuación.
2
y 0 = 6x(y 1) 3 .

Solución.
Separando las variables, tenemos
dy 2
= 6x(y 1) 3
dx
dy
2 = 6xdx.
(y 1) 3
Integrando, tenemos
Z Z
dy
2 = 6xdx + C.
(y 1) 3
Sí: u = y 1 ) du = dy.
Z Z
1
2 du = 6 xdx + C
u3
Z Z
2
u 3 du = 6 xdx + C

1
u3 x2
1 =6 +C
3
2
1
3u 3 = 3x2 + C.

20
Sustituimos u = y 1, tenemos:
1
3 (y 1) 3 = 3x2 + C

1 3x2 + C
(y 1) 3 = .
3
Elevando a la tercera potencia a ambos miembros, tenemos:
h 3x2 + C
1
i3 3
(y 1) 3 =
3
3
3x2 C
y 1= +
3 3
3
C
y = x2 + +1
3
2 3
3 2 C C C
y = x2 + 3 x2 + 3x2 + +1
3 3 3
2 2 3
C x C
y = x6 + Cx4 + + + 1.
3 27

Ejemplo.
Resolver el problema de valor inicial
y
e (1 + y 0 ) = 1, con y(0) = 1.

Solución.
La ecuación escribimos de la siguiente forma
1
1 + y0 = y
e

y 0 = ey 1

dy
= ey 1.
dx
Separando variables, tenemos:
dy
= dx.
ey 1
Integrando
Z Z
dy
= dx + C
ey 1
Z Z
1
dy = dx + C.
ey 1

21
y
Multiplicando y dividiendo por e , la última ecuación, tenemos
Z y Z
1 e
y y
dy = dx + C
e 1 e
Z y Z
e
y
dy = dx + C.
1 e
y y
Sí: u = 1 e ) du = e dy, se tiene
Z Z
du
= dx + C
u

ln juj = x + C

u = ex+C

y
1 e = ex+C .

Así, obtenemos la solución general


y
e =1 ex C.

Utilizando la condición x = 0 y y = 1, tendremos


( 1)
e =1 e0 C

e=1 C

C=1 e.

Por tanto, una solución particular es


y
e = (1 ex ) (1 e).

ECUACIONES HOMOGÉNEAS
Función Homogénea.
La función f (x; y), se llaman función homogénea de grado n, respecto de
las variables x y y, si la función f (x; y) tiene la propiedad de que

f (tx; ty) = tn f (x; y); 8, n y t 2 R.

22
Ejemplo.
Determinar si las funciones son homogéneas o no, e indicar el grado de la
homogeneidad.

a). f (x; y) = x2 3xy + y 2 :

f (tx; ty) = (tx)2 3(tx)(ty)+(ty)2 = t2 x2 3t2 xy+t2 y 2 = t2 (x2 3xy+y 2 ) =


2
t f (x; y).

La función es homogénea de grado dos.


p
b). f (x; y)p= 3 x2 16y 2 . p p p
2 3
f (tx; ty) = 3 (tx)2 16(ty)2 = 3 t2 x2 16t2 y 2 = 3
t2 (x2 16y 2 ) = t 3 x2 16y 2 =
2
t 3 f (xy).

La función es homogénea de grado 23 .


3x2 + y 2
c). f (x; y) = 2
x 2y 2
3(tx)2 + (ty)2 t2 (3x2 + y)2 2
0 (3x + y)
2
f (tx; ty) = = = t = t0 f (x; y).
(tx)2 2(ty)2 t2 (x2 2y)2 (x2 2y)2

La función es homogénea de grado cero.


4y 1
d) f (x; y) = + .
x 2
4(ty) 1 4y 1
f (tx; ty) = + = t0 + = t0 f (x; y).
(tx) 2 x 2

La función es homogénea de grado cero.

e) f (x; y) = x3 + 3xy 2 y 3 + 2.
f (tx; ty) = (tx)3 +3(tx)(ty)2 (ty)3 +2 = t3 (x3 +3xy 2 y 3 +2t 3
) 6= t3 f (x; y).

La función no es homogénea
Ecuación Homogénea.
Una ecuacion diferencial de primer orden
dy
= f (x; y), (1)
dx

se llama ecuación homogénea, si la función f (x; y) es homogénea de grado


cero respecto de x y y.
Método de Solución.
Por hipótesis

23
f (tx; ty) = t0 f (x; y) = f (x; y):

1
Haciendo t = , tenemos
x
y
f (x; y) = f 1; . (2)
x

De la ecuación (1) y (2), obtenemos.


dy y
= f (1; ). (3)
dx x

Ahora efectuamos la sustitición


y
u = o bien y = ux,
x

derivando respecto de x, se tiene


dy du
=u+x . (4)
dx dx

De la ecuación (3) y (4), tenemos


du
u+x = f (1; u),
dx

que es una ecuación de variables separables


du
x = f (1; u) u
dx
du du
= ,
f (1; u) u x

integrando, obtenemos
Z Z
du du
= + C.
f (1; u) u x

y
Finalamente, obtenemos la solución sustituyendo el valor de u por .
x
Ejemplo.
Resolver la ecuación

24
dy 2xy + y 2
=
dx x2
Solución.
La ecuación escribimos en forma (3)
dy 2y y 2
= + .
dx x x

y dy
La ecuación es homogénea de grado cero. Sustituyendo u = y =
x dx
du
u+x en la ecuación, resulta
dx
du
u+x = 2u + u2 .
dx
Separando las variables, se tiene
du
x = u + u2
dx
du dx
= .
u + u2 x
Integrando
Z Z
du dx
= + C1 .
u(1 + u) x
Integrando por fracciones parciales.
Z Z
A B 1
du = dx + C1 . De donde: A = 1, B = 1.
u 1+u x
Z
1 1 R 1
du = dx + C1 , donde C1 = ln jCj.
u 1+u x

ln juj ln j1 + uj = ln jxj + ln jCj

u
ln = ln jCxj
1+u
y
x = Cx
y
1+
x
y
= Cx.
x+y

25
Por tanto, la solución general es

Cx2
y= .
1 Cx
Ejemplo.
Resolver la ecuación
dy
4x 3y + (2y 3x) = 0.
dx
Solución.
La ecuación escribimos en la forma
dy 3y 4x
= .
dx 2y 3x

1
Multiplicado numerador y denominador por , obtenemos
x
y
dy 3 4
= yx ,
dx 2 3
x

y dy du
una ecuación homogénea de grado cero. Sustituyendo u = y = u+x
x dx dx
en la última ecuación, resulta
du 3u 4
u+x = .
dx 2u 3

Separando las variables, se tiene


du 3u 4
x = u
dx 2u 3

du 3u 4 2u2 + 3u
x =
dx 2u 3

du 2u2 + 6u 4
x =
dx 2u 3

2u 3 dx
du = .
2u2 + 6u 4 x

Integrando

26
Z Z
1 2u 3 dx
du = + C1
2 u2 3u + 2 x

1
ln u2 3u + 2 = ln jxj + ln jC1 j
2
1
ln u2 3u + 2 2
= ln jC1 xj.
y
Sustituyendo u = , obtenemos
x
" 1 # 2
y2 y 2
2
3 +2 = [C1 x]
x2 x

y2 3xy + 2x2 2
= [C1 x]
x2

y2 3xy + 2x2 = C1 2 x 2 2
x

donde C = C1 2 . Por tanto, la solución general es

y2 3xy + 2x2 = C .

Ejemplo.
Resolver la ecuación
y
dy x2 sec + y2
= x ,
dx xy

dada la condición inicial y(1) = 0.


Solución.
La ecuación escribimos en forma (3)

dy x y y
= sec +
dx y x x

dy y 1 y y
= sec + .
dx x x x

y dy
La ecuación es homogénea de grado cero. Sustituyendo u = y =
x dx
du
u+x en la última ecuación, resulta
dx

27
du 1
u+x =u sec u + u.
dx

Separando las variables, se tiene


du 1
x =u sec u
dx
du dx
1
=
u sec u x
dx
u cos u du = .
x

Integrando
Z Z
dx
u cos u du = + C.
x

Integrando por partes, tenemos


Z Z
zdv = zv vdz

Z
z = u, dv = cos udu,

dz = du.
v = sen u.
Luego
Z
u sen u sen u du = ln jxj + C

u sen u + cos u = ln jxj + C.

y
Sustituyendo u = , obtenemos la solución general
x
y y y
sen + cos = ln jxj + C.
x x x

Utilizando la condición x = 1 y y = 0, obtenemos

28
1 = C.

Por tanto, la solución particular es


y y y
sen + cos = ln jxj + 1.
x x x

ECUACIONES EXACTAS
Consideremos la ecuación diferencial

M (x; y)dx + N (x; y)dy = 0. (1)

Supóngase que existe una función f (x; y), tal que

@f @f
= M (x; y) ; = N (x; y).
@x @y

Entonces la ecuación (1), se puede escribir como:

@f @f
dx + dy = 0
@x @y
o bien

d [f (x; y)] = 0

y su solucion general es f (x; y) = C.

A la expresión

M (x; y)dx + N (x; y)dy

se llama diferencial exacta y (1) se llama ecuación exacta:


Ejemplo.
Muestre que la ecuación diferencial

2xy 2 dx + 3x2 y 2 dy = 0 ,

es exacta y determine las soluciones.


Solución.
Vemos que el primer miembro de la ecuación es la derivada de la función
f (x; y) = x2 y 3 . Entonces la ecuación dada podemos escribir

29
d x2 y 3 = 0,

En consecuencia, la solución general está dada implicitamente por x2 y 3 = C,


2
y explícitamente por y = k x 3 .

En el ejemplo, planteado no fue difícil veri…car la exactitud de la ecuación


dada, pero en ecuaciones más difíciles no es tan sencillo determinar f (x; y) por
simple inspección. A continuación planteamos un teorema importante, para
determinar la exactitud de una ecuación diferencial.
Teorema (Criterio Para Una Ecuación Exacta).
Supongase que las funciones M (x; y) y N (x; y), y sus derivadas parciales
son continuas en una región rectangular R. Entonces una condición necesaria
y su…ciente para que

M (x; y)dx + N (x; y)dy = 0,

sea exacta en R, es si y sólo si


@M @N
= , 8 (x; y) 2 R.
@y @x

Demostración. El teorema demostraremos en dos partes:


Condición Necesaria:
Para simpli…car la condición necesaria, consideremos que existe una función
f (x; y) tal que se cumple

@f @f
M (x; y)dx + N (x; y)dy = dx + dy
@x @y

donde
@f @f
M (x; y) = y N (x; y) = .
@x @y

Derivando parcialmente la primera ecuación respecto de y, tenemos

@M @f @f @2f @ @f @N
= = = = , 8 (x; y) 2 R.
@y @y @x @y@x @x @y @x

30
Condición Su…ciente:
Esta condición demostramos en la siguiente sección, que consiste en con-
struir una función f (x; y), en realidad este hecho no es más que el método de
solución.

Método de Solución.
Sea la ecuación

M (x; y)dx + N (x; y)dy = 0

tal que
@M @N
= ;
@y @x

@f @f
donde = M (x; y), = N (x; y).
@x @y
Integrando la primera ecuación con respecto de x mientras y se mantiene
como constante, se tiene.
R
f (x; y) = M (x; y)dx + g(y). (2)

La función arbitraria g(y) es la "constante de integración" . Derivado (2)

@f
respecto de y, y teniendo en cuenta que = N (x; y) resulta.
@y
@f @ R
= M (x; y)dx + g0(y) = N (x; y)
@y @y

luego
@ R
g 0 (y) = N (x; y) M (x; y)dx.
@y

Integrando respecto de y, obtenemos


R @ R
g(y) = N (x; y) M (x; y)dx dy,
@y

…nalmente obtenemos la solución general, sustituyendo el valor de g(y)


en (2), la solución es de la forma:

31
f (x; y) = C.

Por otra parte si se toma la ecuación


@f
= N (x; y),
@y
integrando respecto de y, y derivando el resultado, obtenemos en forma
análoga

R R @ R
f (x; y) = N (x; y)dy + h(x) y h(x) = M (x; y) N (x; y)dy dx .
@x

Ejemplo.
Resolver la ecuación

x(2x2 + y 2 )dx + y(x2 + 2y 2 )dy = 0.

Solución.
Probaremos si esta ecuación dada es exacta, siendo

M (x; y) = x(2x2 + y 2 ), N (x; y) = y(x2 + 2y 2 ).

Derivando
@M @N
= 2xy, = 2xy
@y @x
como
@M @N
= .
@y @x
La ecuación dada es exacta, donde
@f @f
= M (x; y) = x(2x2 + y 2 ) y = N (x; y) = y(x2 + 2y 2 ).
@x @y

Integrando la primera ecuación respecto de x tenemos


f (x; y) =
R
(2x3 + xy 2 )dx + g(y)

1 4 x2 2
f (x; y) = x + y + g(y). (3)
2 2

32
@f
Derivando (3) respecto de y, y como = N (x; y), resulta
@y
@f
= x2 y + g 0 (y) = y(x2 + 2y 2 ).
@y

Luego

g 0 (y) = 2y 3 .

Integrando
R 1 4
g(y) = 2y 3 dy = y .
2

Sustituyendo el valor de g(y) en (3), obtenemos


1 4 1 2 2 1 4
f (x; y) = x + x y + y .
2 2 2

Multiplicando la ecuación anterior por 2, por tanto la solución general es:

x4 + x2 y 2 + y 4 = C.

Ejemplo
Resolver la ecuación

(y + y cos xy)dx + (x + x cos xy)dy = 0.

Solución.
Siendo

M (x; y) = y + y cos xy y N (x; y) = x + x cos xy.

Probemos si la ecuación inicial es exacta


@M @N
= 1 + cos xy xy sen xy, = 1 + cos xy xy sen xy,
@y @x

como son iguales


@M @N
= .
@y @x

33
La ecuación dada es exacta, donde
@f @f
= M (x; y) = y + y cos xy y = N (x; y) = x + x cos xy.
@x @y

Integrando la primera ecuación respecto de x tenemos


R
f (x; y) = (y + y cos xy)dx + g(y)

f (x; y) = xy + sen xy + g(y). (4)

@f
Derivando (4) respecto de y y como = N (x; y), resulta
@y
@f
= x + x cos xy + g 0 (y) = x + x cos xy
@y

g 0 (y) = 0.

Integrando

g(y) = C1 .

Sustituyendo el valor de g(y) en (4), obtenemos

f (x; y) = xy + sen xy + C1 .

Por lo tanto, la solución general es:

xy + sen xy = C.

FACTOR INTEGRANTE
Consideremos que la ecuación

M (x; y) dx + N (x; y) dy = 0, (1)

no es exacta. Muchas veces se logra hallar una función (x; y), tal que
multiplicada todos los términos de la ecuación (1), por esta función se convierte
en una ecuación diferencial exacta. La función (x; y), se llama factor integrante

de la ecuación (1).
Para determinar un factor integrante de (1), multiplicamos los dos miembros
de (1), por la función , esto es

34
M dy+ N dx = 0.

Esta ecuación será exacta si cumple la siguiente condición:

@( M ) @( N )
= .
@y @x
Aplicando la regla del producto de la derivada, se tiene
@M @ @N @
+M = +N
@y @y @x @x

1 @ @ @M @N
N M = . (2)
@x @y @y @x

Supóngase que la ecuación (1), admite un factor integrante que depende


@ @ d
solamente de x, esto es, = (x), entonces =0y = , de modo que
@y @x dx
(2) se reduce a :

@M @N
1d @y @x
=
dx N
o bien
0 1
@M @N
d B @y @x C
=B
@
C dx.
A
N

Hacemos
@M @N
@y @x
g (x) = .
N

Siendo
d
= g(x)dx.

Integrando

35
Z Z
1
d = g(x)dx

Z
ln j j = g(x)dx.

En consecuencia, el factor integrante de (1), que depende sólo de x es:


R
g(x)dx
(x) = e .

En forma análoga podemos obtener, el factor integrante de (1) que depende


solamente de y, esto es = (y), siendo

@N @M
@x @y
h(y) =
M

Por tanto, el factor integrante de la ecuación (1) que depende solamente de


y es:
R
g(y)dy
(y) = e .

Ejemplo.
Resolver la ecuación

2x2 y + 2y + 5 dx + 2x3 + 2x dy = 0. (4)

Solución.
La ecuación dada ni es separable, ni homogénea, veamos si es exacta. Donde:

M (x; y) = 2x2 y + 2y + 5 y N (x; y) = 2x3 + 2x

@M @N
= 2x2 + 2 , = 6x2 + 2.
@y @x
Como
@M @M
6= ,
@y @y

La ecuación no es exacta. Ahora veamos si (4) admite un factor integrante


que depende sólo de x, esto es:

36
2x2 + 2 6x2 2 4x2 2x
g(x) = = = 2 .
2x3 + 2x 2x (x2 + 1) (x + 1)

Luego, el factor integrante es


Z
2x
x2 +1
dx 1
lnjx2 +1j 2 1
(x) = e =e = elnjx +1j
= x2 + 1

1
(x) = .
x2 + 1

Multiplicando la ecuación (4), por este factor integrante se tiene:

2x2 y + 2y + 5 2x3 + 2x
dx + dy = 0
x2 + 1 x2 + 1

2y(x2 + 1) 5 2x(x2 + 1)
+ dx + dy = 0
x2 + 1 x2 + 1 x2 + 1

5
2y + dx + 2xdy = 0. (5)
x2 +1

Donde:
5
M (x; y) = 2y + y N (x; y) = 2x
x2 +1

@M @N
=2= .
@y @x

Esto muestra que la ecuación (5) es exacta. De manera que:

@f 5 @f
= 2y + 2 ; = 2x.
@x x +1 @y

Integrando la primera ecuación respecto de x, tenemos:


Z
5
f (x; y) = 2y + 2 dx + g(y) = 2xy + 5 arctan x + g(y).
x +1

@f
Derivando f (x; y) respecto de y, y como = N (x; y) resulta :
@x

37
@f
= 2x + g 0 (y) = 2x
@y

g 0 (y) = 0.

Integramos

g(y) = C.

Por tanto, la solución general es:

2xy + 5 arctan x + C = 0.

Ejemplo.
Resolver la ecuación
y + xy 2 dx xdy = 0. (6)
Solución.
Donde:

M (x; y) = y + xy 2 y N (x; y) = x

@M @N
= 1 + 2xy = 1
@y @x

@M @N
6= .
@y @x

La ecuación dada no es exacta. Veamos si (6) admite un factor integrante


respecto de x
@M @N
@y @x 1 + 2xy + 1 2xy + 2 2(xy + 1)
g(x) = = = = ,
N x x x

esta función no depende solamente de x. Ahora veamos si admite un factor


integrante respecto de y
@N @M
@x @y 1 1 2xy 2 2xy 2 2xy
h(y) = = 2
= 2
= =
M y + xy y + xy y + xy 2

2(1 + xy) 2
=
y(1 + xy) y

38
La función h(y) es solamente de y. El factor integrante
R
es R 1
h(y)dy 2 y dy 2 lnjyj
(y) = e =e =e =
2
elnjyj
1
(y) = .
y2

Multiplicamos la ecuación (6) por el factor integrante

(y + xy 2 ) x
dx dy = 0,
y2 y2
o bien
1 x
+ x dx dy = 0. (7)
y y2
Donde:
1 x
M (x; y) = +x y N (x; y) =
y y2

@M 1 @N
= = .
@y y2 @x

La ecuación (7) es exacta. De manera que:


@f 1 @f x
= + x; = .
@x y @y y2
Integrando la primera ecuación respecto de x, tenemos
Z
1 x x2
f (x; y) = + x dx + g(y) = + + g(y).
y y 2

@f
Derivando f (x; y) respecto de y, y como = N (x; y), resulta
@y
@f x x
= + g 0 (y) =
@y y2 y2

g 0 (y) = 0.

Integrando

g(y) = C.

39
Por tanto, la solución general es

x x2
+ = C.
y 2

ECUACIONES LINEALES
Una ecuación de la forma
dx
+ P (x)y = Q(x), (1)
dy

se llama ecuación diferencial lineal de primer orden donde P (x) y Q(x)


funciones contínuas de x (o constantes) en el intervalo I.
Método de Solución.
La ecuación (1), escribimos en forma diferencial

[P (x)y Q(x)] dx + dy = 0. (2)

Veamos la exactitud de esta ecuación


@M @N
= P (x); = 0.
@y @x

La ecuación (2), no es exacta. Sin embargo, se puede obtener sencillamente


un factor integrante (x) que depende solamente de x. Efectuamos

[ (x)P (x)y (x)Q(x)] dx + (x)dy = 0.

La ecuación será exacta, si se cumple


@M @N d
= (x)P (x); = .
@y @x dx

De manera que
d
= (x)P (x).
dx

Es una ecuación de variables separables


d
=P(x)dx
Integrando:

40
Z Z
d
= P (x)dx

Z
ln j j = P (x)dx.

Por tanto el factor integrante de la ecuación (1), es:


R
P (x)dx
(x) = e .

Ahora multiplicaremos la ecuación (1), por este factor integrante


R R R
P (x)dx dy P (x) P (x)
e +e p(x)y = e Q(x).
dx

El primer miembro de esta ecuación es la derivada del producto del factor


integrante y y:
d h R P (x) i R
e y = e P (x)dx Q(x).
dx

Integrando
R
Z R
P (x) P (x)dx
e y= e Q(x)dx + C.

Por tanto la solución general de (1), es:


R
Z R
y = e P (x)dx e P (x)dx Q(x)dx + C .

Ejemplo.
Resolver la ecuación
dy
x2 + 4 + xy = 0. (3)
dx
Solución.
La ecuación escribimos en la forma estándar
dx x
+ 2 y = 0. (4)
dy x +4
Siendo
x
P (x) = y Q(x) = 0.
x2 +4

41
El factor integrante es.
R 1R 2x 2 2
1

= e 2 lnjx +4j
= elnjx +4j 2
dx 1 1
P (x)dx
(x) = e = e2 x2 +4 = (x2 + 4) 2 .

Luego multiplicamos la ecuacion (4), por el factor integrante


1
1 dy x(x2 + 4) 2
(x2 + 4) 2 + y=0
dx x2 + 4

1 dy 1
(x2 + 4) 2 + x(x2 + 4) 2 y = 0.
dx

Resulta
d h 2 1
i
(x + 4) 2 y = 0.
dx

Integrando
1
(x2 + 4) 2 y = C.

Por tanto la solución general es:


C C
y= 1 =p .
(x2 + 4) 2 x2 + 4

Ejemplo.
Resolver la ecuación:
dy x sen x y
= .
dx x
Solución.
La ecuación escribimos en la forma de (1):

dy 1
+ y = sen x (5)
dx x
1
La ecuación (5) es lineal, con P (x) = ; y Q(x) = sen x.
x
El factor integrante es

R 1
dx
(x) = e x = elnjxj = x.

Ahora multiplicaremos (5) por el factor integrante

42
dy
x + y = x sen x.
dx
De manera que
d
[xy] = x sen x.
dx
Integrando
Z Z
xy = x sen xdx + C = x cos x + cos dx + C = x cos x + sen x + C.
u = x, dv = Zsen xdx,
du = dx. v= sen xdx,

v=
cos x .
Por tanto, la solución general es:
sen x C
y= cos x + + ; para x > 0.
x x

Ejemplo.
Resolver la ecuación

(cos2 x sen x)dy + (y cos3 x 1)dx = 0.

Solución.
La ecuación escribimos en la forma de (1)

dy cos3 x 1
+ y=
dx cos2 x sen x cos2 x sen x
dy cos x 1
+ y=
dx sen x cos2 x sen x
o bien
dx 1
+ (cot x) y = 2
. (6)
dy cos x sen x

La ecuación (6), es una ecuación lineal con P (x) = cot x. El factor integrante
es:
Z
cot xdx
(x) = e = elnjsen xj = sen x.

43
Ahora multiplicamos (6) por el factor integrante

dy cos x sen x
sen x + sen x y= 2
dx sen x cos x sen x
o bien
dy 1
sen x + (cos x) y = .
dx cos2 x
De manera que
d 1
[(sen x) y] = .
dx cos2 x
Integrando
Z
1
(sen x) y = + C = tan x + C.
cos2 x

Por tanto la solución general es:

y = sec x + C csc x.

TEMA No 3

ECUACIONES DIFERENCIALES LINEALES DE


SEGUNDO ORDEN

INTRODUCCIÓN
Una ecuación diferencial lineal de segundo orden en su forma canónica es de
la siguiente forma:

d2 y dy
2
+ P (x) + Q(x)y = R(x)
dx dx

o bien:

y 00 + P (x)y 0 + Q(x)y = R(x) (1)

donde P (x), Q(x) y R(x) son funciones continuas sobre un intervalo I. Si


R(x) 6= 0 en (1), se llama ecuación no homogénea. Si R(x) = 0 en (1), es de la
forma:

44
y 00 + P (x)y 0 + Q(x)y = 0

se llama ecuación homogénea.

Ejemplo.

1. La ecuación y 00 + x2 y 0 + xy =ex , es una ecuación de 2do. orden no


homogénea.

2. La ecuación: y 00 + (sen x)y 0 + y = 0, es una ecuación homogénea.

Teorema (Teorema de Existencia y Unicidad de Una Solución Única).


Supongase que P (x), Q(x) y R(x) son funciones continuas sobre el intervalo
I, y que contienen al punto x0 :Entonces existe una solución única y solo uno
y(x) de la ecuación (1) sobre el intervalo completo I y las condiciones iniciales

y(x0 ) = y0 , y 0 (x0 ) = y00 :

Ejemplo.
Muestre que la función y = sen x, es una solución de la ecuación homogénea
y 00 + y = 0 que satisface los valores iniciales y(0) = 0, y 0 (0) = 1.

Solución.
Veri…camos que y = sen x, evidentemente es una solución de la ecuación
dada , además y = cos x, más en general y = C1 sen x + C2 cos x, donde C1
y C2 , son constantes arbitrarias y también son soluciones. Pero simplemente
y = sen x satisface los valores iniciales, puesto que: y = sen x:
Si: y(0) = 0 , entonces sen(0) = 0:
Luego, si y = sen x =) y 0 = cos x:
Si: y 0 (0) = 1 , entonces cos(0) = 1:
Esto muestra que la función y = sen x es la única solución de la ecuación
y 00 + y = 0, que satisface las condiciones iniciales dadas.
Teorema A.
Sea y1 (x) y y2 (x) dos soluciones linealmente independientes de la ecuación
homogenea:

y 00 + P (x)y 0 + Q(x)y = 0. (2)

En un intervalo [a; b] entonces la combinación lineal

y = C1 y1 (x) + C2 y2 (x),

45
se llama "solución general " de la ecuación (2), para todo par de constantes
C1 y C2 .
Demostración.
Sea y(x), cualquier solución de la ecuación (2) sobre el intervalo [a; b]. Ahora
debemos probar que es posible hallar las constantes C1 y C2 tal que: y(x) =
C1 y1 (x) + C2 y2 (x); 8x 2 [a; b]. Todo queda completamente determinado por
el valor y(x0 ) y el valor de su derivada y 0 (x0 ) en un único punto x0 . Esto es:

C1 y1 (x0 ) + C2 y2 (x0 ) = y(x0 )


C1 y10 (x0 ) + C2 y20 (x0 ) = y 0 (x0 ):

Este sistema de ecuaciones tiene una solución única, para las constantes C1
y C2 , siempre que la determinante de los coe…cientes sea diferente de cero, es
decir:
y1 (x0 ) y2 (x0 )
= y1 (x0 )y20 (x0 ) y2 (x0 )y10 (x0 ) 6= 0.
y10 (x0 ) y20 (x0 )

La determinante de…nida por

y1 y2
W (y1 ,y2 ) = = y1 y20 y2 y10 ,
y10 y20

se conoce como el "wronskiano" de y1 y y2 :

Corolario.
Dos soluciones y1 (x) y y2 (x) de la ecuación homogénea

y 00 + P (x)y 0 + Q(x)y = 0,

en un intervalo [a; b], son linealmente independientes, si y sólo si, su wron-


skiano es diferente de cero.

Ejemplo.
Dada la ecuación lineal homogénea

y 00 + 4y = 0.

a). Muestre que y1 = sen 2x y y2 = cos 2x son dos soluciones linealmente


independientes.
b). Hallar la solución general.
c). Hallar la solución particular para los valores iniciales y(0) = 4 , y 0 (0) = 3.

Solución.
a). Las soluciones y1 = sen 2x y y2 = cos 2x, serán linealmente independi-
entes si su wronskiano es diferente de cero, es decir:

46
sen 2x cos 2x
W (y1 , y2 ) = = 2 sen2 2x 2 cos2 2x = 2(sen2 2x+
2 cos 2x 2 sen 2x

cos2 2x) = 2 6= 0.
Esto muestra que evidentemente y1 y y2 son soluciones linealmente inde-
pendientes.
b). La combinación lineal de las soluciones dadas es la solución general, esto

es

y = C1 sen 2x + C2 cos 2x.

c). Para obtener la solución particular resolvemos el sistema:

C1 sen 2x + C2 cos 2x = y;
2C1 cos 2x 2C2 sen 2x = y10 :

Aplicando las condiciones iniciales en el sistema, resulta

C1 sen 2(0) + C2 cos 2(0) = 4;


2C1 cos 2(0) 2C2 sen 2(0) = 3:

3
De donde las soluciones son: C1 = y C2 = 4. En consecuencia, la solución
2
particular buscada es:
3
y= sen 2x + 4 cos 2x.
2

REDUCCIÓN DE ORDEN.
Para desarrollar este procedimiento, supongase que y1 (x) es una solución no
nula conocida de la ecuación homogénea

y 00 + P (x)y 0 + Q(x)y = 0. (1)

De hecho Cy1 (x), donde C es una constante, también es una solución de (1).
Ahora la idea básica es sustituir C por una función desconocida digamos v(x)
e intentar hallar v(x), tal que y2 (x) = v(x)y1 (x) sea la solución de la ecuación
(1).
Como suponemos que y2 = vy1 es una solución de la ecuación diferencial
(1), entonces debe cumplirse que

y200 + P (x)y20 + Q(x)y2 = 0. (2)

47
Las derivadas de y2 = vy1 son

y20 = vy10 + v 0 y1

y200 = vy100 + v 0 y10 + v 0 y10 + v 00 y1 = vy100 + 2v 0 y10 + v 00 y1

Sustituyendo estas expresiones en (2), tenemos

(vy100 + 2v 0 y10 + v 00 y1 ) + P (vy10 + v 0 y1 ) + Qvy1 = 0,

reordenando los terminos queda

v (y100 + P y10 + Qy1 ) + y1 v 00 + (2y10 + P y1 )v 0 = 0

como y1 es una solución de (1), entonces se reduce a

y1 v 00 + (2y10 + P y1 )v 0 = 0.

Hacemos la sustitución u = v 0 , entonces se reduce a

y1 u0 + (2y10 + P y1 )u = 0.

una ecuación diferencial lineal de primer orden y de variables separables


u0 y10
= 2 P
u y1

Integrando:
Z Z Z
1 y10
du = 2 dx P dx
u y1
Z
ln juj = 2 ln jy1 j P dx
Z
P dx
2
u =elnjy1 j e .

Como u = v 0 , se tiene
Z
P dx
v 0 = y1 2 e .

Intengrando nuevamente, resulta

48
Z
Z P (x)dx
1
v(x) =
y12
e dx.

Por tanto la segunda solución linealmente independiente de la ecuación (1)


es:

y2 = vy1
Z
Z P (x)dx
1
y2 = y1
y12
e dx.

Ejemplo.
La función y1 = x2 es una solución de la ecuación

x2 y 00 3xy 0 + 4y = 0,

determinar la segunda solución linealmente independiente y la solución gen-


eral.
Solución.
Escribimos la ecuación en su forma canónica
3 0 4
y 00 y + 2 y = 0.
x x
3
Como P (x) = , la segunda solución viene dado por y2 = vy1 . Deter-
x
minemos v(x):
Z Z
Z 3 Z 1 Z
1 dx 1 3 x dx 1 3 lnjxj
v(x) = e x dx = e dx = e dx =
2 2
[x ] x4 x4
Z Z
1 3 1
x dx = dx = ln jxj.
x4 x

Luego la segunda solución es

y2 = x2 ln jxj.

Por tanto, la solución general es

y = C1 y1 (x) + C2 y2 (x)

y = C1 x2 + C2 x2 ln jxj.

49
Ejemplo.
sen x
La función y1 = p es una solucion de
x

1
x2 y 00 xy 0 + x2 y = 0, en el intervalo (0; ),
4
determine una segunda solución linealmente independiente.
Solución.
La ecuación en su forma canónica es:
1 0 1
y 00 + y + 1 y = 0.
x 4x2
1
Como P (x) = la segunda solución viene dado por y2 = vy1 . Determinemos
x
v(x): Z
Z 1 Z Z
1 dx 1 1
x elnjxj 1 dx =
2e 2 e
lnjxj
v(x) = dx = dx =
sen x sen x 2
sen x
p
x x x
Z Z
x 1 1
x dx = dx
sen2 x R sen2 x
= csc2 xdx = cot x.

Luego la segunda solución es


sen x cos x sen x cos x
y2 = ( cot x) p = p = p ,
x sen x x x

cos x
ó simplemente y2 = p .
x
Por tanto, la solución general es
sen x cos x
y = C1 p + C2 p .
x x
Ejemplo.
La función y1 = x3 es una solución de la ecuación

x2 y 00 (2x2 + 6x)y 0 + (6x + 12)y = 0.

Hallar la segunda solución linealmente independiente y la solución general.


Solución.
La ecuación escribimos de la forma
6 6x + 12
y 00 2+ y0 + y = 0.
x x2

50
6
Como P (x) = 2+ la segunda solución está dado por y2 = vy1 .
x
Determinemos v(x): Z
! !
Z 6 Z R 6 R R 1
1 2+
x
dx 1 2+ dx R 1 2 dx+6 dx
2e e e
v(x) = dx = x dx = x dx
[x3 ] x6 x6
Z Z Z Z
1 2x+6lnjxj 1 2x lnjxj6 e2x 6
=
x6
e dx =
x6
e e dx = x6
x dx = e2x dx =
e2x
.
2
x3 e2x
Como y2 = vy1 ;entonces y2 = ,
2
o simplemente y2 = x3 e2x .
Por tanto la solución general es

y = C1 x3 + C2 x3 e2x .

ECUACIONES LINEALES HOMOGÉNEAS CON COEFICIENTES


CONSTANTES

Sea la ecuación lineal homogénea de segundo orden

y 00 + P (x)y 0 + Q(x)y = 0.

Consideremos el caso en el que P (x) y Q(x), son constantes, es decir

y 00 + py10 + qy = 0. (1)

donde los coe…cientes p y q son constantes reales. La ecuación (1) se llama


ecuación lineal homogénea con coe…cientes constantes. Supongase que la función
exponencial

y =emx , (2)

donde m es una constante, es una posible solución de la ecuación (1), cuyas


derivadas son

y 0 = memx

y 00 = m2 emx .

51
Sustituyendo (2) y sus derivadas en (1), tenemos

m2 emx + pmemx + q emx = 0

(m2 + pm + q)emx = 0.

Esta ecuación se satisface solamente cuando la ecuación caracteristica o


ecuación auxiliar sea cero. Es decir

m2 + pm + q = 0.

Las raices m1 y m2 se obtienen mediante la fórmula general

p
p p2 4q
m= .
2

Ahora debemos analizar, al igual como en álgebra los casos en el que el


discriminante p2 4q, sea positiva, cero y negativo.

Raices Reales y Distintas.


Si, p2 4q > 0 , entonces m1 y m2 son raices reales y distintas, y las soluciones
son:

y1 =em1 x , y2 =em2 x .

Ademas y1 y y2 son linealmente independientes, puesto que:


y1 em1 x
= m2 x =e(m1 m2 )x
6= Ctte::
y2 e

Por tanto la solución general de la ecuación (1) es

y = C1 em1 x + C2 em2 x .

Raices Reales e Iguales.


Si p2 4q = 0, entonces m1 y m2 son raices reales e iguales, es decir m1 = m2
P
y obtenemos solamente una sola solución: y1 =em1 x con m1 = .
2
Sin embargo, podemos obtener la segunda solución …nalmente independiente
utilizando el método de "reducción de orden"
Sea: y1 =e( 2 )x , siendo
P

52
Z R
1
v=
y12
e P (x)dx
dx =
Z R
Z Z
1 1
h P
i2 e p dx
dx =
e px
e px
dx = dx = x.
e 2 x

De manera que

y2 = vy1 =) y2 = xem1 x .

Por tanto, la solución general de la ecuación (1) es

y = C1 em1 x + C2 xem2 x .

Raices Complejas.
Si p2 4q < 0, entonces las raices m1 y m2 son raices complejas conjugadas
y escribimos p
m1 = + i y m2 = i ; donde i = 1; y 2 R:
p
4q p2
Con = p2 y = 2 .
Luego las soluciones son:

y1 =e( +i )x
, y2 =e( i )x
.

Pero, estamos interesados solamente en soluciones de funciones reales, para


ello consideremos la fórmula de Euler

ei = cos + i sen , 2 R.

Hacemos

em x =e(1 +i )x
=e x
ei x
=e x
(cos x + i sen x),

em x =e(
2 i )x
=e x
e i x
=e x
(cos x i sen x),

donde:

cos( x) = cos x, sen( x) = sen x.

53
De manera que:
em1 x + em2 x e x
(cos x + i sen x) + e x
(cos x i sen x)
y1 = =
2 2
x x
e cos x + i e sen x + e x cos x i e x sen x 2e x cos x
= = =e x cos x.
2 2
em1 x em2 x e x (cos x + i sen x) e x (cos x i sen x) 2i e x sen x
y2 = = = =e x
sen x.
2i 2i 2i

Por tanto la solución general de la ecuación (1) es

y = C1 e x
cos x + C2 e x
sen x.

Ejemplo.

Hallar la solución general de la ecuación:

y 00 + y 0 6y = 0

Solución.
Es una ecuación homogénea de coe…cientes constantes. La ecuación carac-
terística de la ecuación inicial es

m2 + m 6=0

(m + 3)(m 2) = 0

m + 3 = 0, m 2=0

m1 = 3, m2 = 2.

Como las raices son reales y distintas, las soluciones son:

y1 =e 3x
, y2 =e2x

Además y1 y y2 son linealmente independientes puesto que:

y1 e 3x
= 2x =e 3x 2x
=e 5x
6= Ctte:
y2 e

Por tanto la solución general es:

54
y = C1 e 3x
+ C2 e2x .

Ejemplo.

Hallar la solución general de la siguiente ecuación:

y 00 2y 0 y=0

Solución.
Es una ecuación homogénea de coe…cientes constantes. La ecuación carac-

terística es

m2 2m 1=0
q
2 p p
( 2) ( 2) 4( 1) 2 8 2 2 2 p
m= = = =1 2.
2 2 2

Luego: p
m1 = 1 p2,
m2 = 1 2,

Como las raices son reales y distintas, de modo que las soluciones son
p p
y1 =e(1+ 2)x
, y2 =e(1 2)x
.

Vemos si son linealmente independientes y1 y y2


p
y1 e(1+ 2)x p p p
= p =e[(1+ 2)x] [(1 2)x]
=e2 2x
6= Ctte:
y2 e(1 2)x

Por tanto, la solución general es


p p
y = C1 e(1+ 2)x
+ C2 e(1 2)x
.

Ejemplo.

Hallar la solución general de la ecuación

55
y 00 + 4y 0 + 4y = 0.

Solución.
La ecuación caracteristica es

m2 + 4m + 4 = 0

(m + 2)2 = 0

m+2=0

m= 2.

Siendo m1 = m2 = 2, raices reales e iguales, las soluciones son:

y1 =e 2x
, y2 = xe 2x
.

Por tanto, la solución general es

y = C1 e 2x
+ C2 xe 2x
.

Ejemplo.
Determinar la solución general de la ecuación

y 00 + y 0 + y = 0.

Solución.
La ecuación característica de la ecuación dada es

m2 + m + 1 = 0.

Aplicando la fórmula general, tenemos


p p
1 1 4 1 3 1 1p p 1 1p
m= = = 3 1= 3i,
2 2 2 2 2 2

Son raices complejas y conjugadas, es decir


p p
1 3i 1 3i
m1 = + y m2 = .
2 2 2 2

56
p
1 3
Donde = y = , entonces las soluciones son
2 2
p
3
y1 =e cos x =e 2 cos
1
x x
x,
2
p
3
y2 =e sen x =e
1
x x
2 sen x.
2

Por tanto, la solución general es:

y = C1 e x
cos x + C2 e x
sen x
p p
3 3
y = C1 e x + C2 e
1 1
2x cos 2x sen x.
2 2

MÉTODO DE LOS COEFICIENTES INDETERMINADOS

Consideremos la ecuación diferencial lineal no homogenea

y 00 + P (x)y 0 + Q(x)y = R(x). (1)

Estudiamos para aquellos casos que se conoce la solución general yg (x) de


la correspondiente ecuación homogenea

y 00 + P (x)y 0 + Q(x)y = 0. (2)

Además, si yp (x) es cualquier solución particular de (1), entonces

y = yg (x) + yp (x),

Es la solución general de la ecuación (1). El método de los coe…cientes


indeterminados es un procedimiento que nos permite hallar la solución particular
yp , cuando la ecuación (1) tiene la forma

y 00 + py 0 + qy = R(x) (3)

donde p; q son constantes y R(x) es una función exponencial (eax ), un poli-


nomio
(a0 xn + a1 xn 1 + ::::::: + an 1 x + an ), una función trigonométrica como
(cos x ; sen x); o una combinación lineal de tales funciones

57
cos x
R(x) = (a0 xn + a1 xn 1
+ ::::::: + an 1x + an )eax .
sen x

Si R(x) = ln x, R(x) = x1 , R(x) = tan x, :::, etc. el método no funciona.

Primero, analizaremos la ecuación (3) de la forma:

y 00 + py 0 + qy =eax . (4)

Supongase que la función exponencial

yp = Aeax , (5)

es una posible solución particular de la ecuación (4), donde A es el coe…ciente


indeterminado que debe calcularse. Cuyas derivadas de (5) son:

yp0 = Aaeax ; yp00 = Aa2 eax .

Sustituyendo (5) y sus derivadas en (4), tenemos

Aa2 eax + pAaeax + qAeax =eax

factorizando

A(a2 + pa + q)eax =eax

1
A= . (6)
a2 + pa + q

Luego sustituyendo el valor de A en la ecuación (5), obtenemos la solución


particular, excepto cuando el denominador de (6) sea cero, esto ocurre cuando
a es una raíz de la ecuación característica:

m2 + pm + q = 0.

Ahora intentamos de la forma

yp = Axeax . (7)

58
Siendo sus derivadas

yp0 = Aeax + Aaxeax

yp00 = Aaeax + Aaeax + Aa2 xeax = 2Aaeax + Aa2 xeax .

Sustituyendo (7) y sus derivadas en (4), resulta

2Aaeax + Aa2 xeax + p(Aeax + Aaxeax ) + qAxeax =eax .

2Aaeax + Aa2 xeax + pAeax + pAaxeax + qAxeax =eax

A(a2 + pa + q)xeax + A(2a + p)eax =eax

Como a es una raiz de (4), entonces se reduce a

A(2a + p)eax =eax

A(2a + p) = 1

1
A= .
2a + p

Sustituyendo, el valor de A en la ecuación (7), obtenemos la solución partic-


p
ular, salvo cuando a = . Finalmente intentamos de la forma
2
yp = Ax2 eax (8)

Siendo

yp0 = 2Axeax + Aax2 eax

yp00 = 2Aeax + 2Aaxeax + 2Aaxeax + Aa2 x2 eax = 2Aeax + 4Aaxax + Aa2 x2 eax .

Sustituyendo (8) y sus derivadas en la ecuación (4), tendremos

A(a2 + pa + q)x2 eax + 2A(2a + p)xeax + Aax2 eax =eax

2Aeax =eax

de manera que

59
1
A= .
2

Ahora analizamos la ecuacion (3) de la forma

y 00 + py 0 + qy = a0 xn + a1 xn 1
+ + an 1x + an , (9)

es natural suponer que la solución particular también es un polinomiode


grado n

yp = A0 xn + A1 xn 1
+ + An 1x + An ; (10)

Sustituyendo (10) en (9) y sus derivadas e igualando los coe…cientes de


potencias iguales de x obtenemos los coe…cientes indeterminados A0 ; A1 ; : : : ; An :
El procedimiento falla cuando q = 0, entonces intentamos

yp = x(A0 xn + A1 xn 1
+ : : : + An 1x + An ).

En el caso particular si p = 0 y q = 0, la solución obtenemos por integración


directa.
Finalmente, analizaremos cuando la ecuación (3) es de la forma

y 00 + py 0 + qy = sen x (11)

Supongase que, la solución particular de la ecuación (11) es

yp = A cos x + B sen x (12)

Sustituyendo (12) y sus derivadas en (11) e igualando los coe…cientes de


cos x y sen x obtenemos los coe…cientes indeterminados A y B. El proced-
imiento falla cuando cos x; o sen x es una solución de la ecuación (10), o bien
i es una raiz de la ecuación caracteristica. Entonces intentamos de la forma:

yp = x(A cos x + B sen x)

= Ax cos x + Bx sen x.

Ejemplo.

Hallar la solución general de la ecuación

60
y 00 2y 0 3y = 4e 3x

Solución.
Primero determinamos la solución general de la ecucación homogénea.

y 00 2y 0 3y = 0:

La ecuación característica es

m2 2m 3 = 0
(m 3)(m + 1) = 0.

Siendo las raices

m1 = 3; m2 = 1.

De modo que la solución general es

yg = C1 e3x + C2 e x
.

Ahora determinamos la solución particular yp . Como a = 3 es diferente de


m1 y m2 entonces planteamos de la forma

yp = Ae 3x

yp0 = 3Ae 3x

yp00 = 9Ae 3x

Sustituyendo, estos valores en la ecuaciones no homogeneas dada, tenemos

9Ae 3x
2( 3Ae 3x
) 3Ae 3x
= 4e 3x

9Ae 3x
+ 6Ae 3x
3Ae 3x
= 4e 3x

12Ae 3x
= 4e 3x

1
A=
3

Luego, la solución particular es


1
yp =
3
e 3x
.

61
Por tanto la solución general es
1
y = yg + yp = C1 e3x + C2 e x
+ e 3x
.
3

Ejemplo.

Hallar la solución general de la ecuación

y 00 + 2y 0 + y = 3e x
. (1)

Solución.
Primero determinamos la solución general yg de la ecuación homogénea

y 00 + 2y 0 + y = 0 (2)

La ecuación característica es

m2 + 2m + 1 = 0

(m + 1)2 = 0

son raices reales y repetidas

m1 = m2 = 1

En consecuencia la solución general de la ecuación (2) es

yg = C1 e x
+ C2 xe x
.

Ahora determinamos la solucion particular yp de la ecuacion (1). Como


a = 1 es una raíz repetida de la ecuación característica, entonces buscamos la
solución particular de la forma

yp = Ax2 e x
.

Las derivadas son

yp0 = 2Axe x
Ax2 e x

yp00 = 2Ae x
2Axe x
2Axe x
+ Ax2 e x

yp00 = 2Ae x
4Axe x
+ Ax2 e x

62
Sustituyendo yp y sus derivadas en la ecuación (1), se tiene

2Ae x
4Axe x
+ Ax2 e x
+ 4Axe x
2Ax2 e x
+ Ax2 e x
= 3e x

2Ae x
= 3e x
.

Igualamos los coe…cientes, obtenemos.

2A = 3
3
A= .
2

En consecuencia la solución particular de la ecuación (1) es

yp = Ax2 e x

3 2
yp = x e x
.
2

Por tanto la solución general de (1) es:

y = yg + yp

3
y = C1 e x
+ C2 xe x
+ x2 e x
2
Ejemplo.

Hallar la solución general de la ecuación

y 00 3y 0 + 2y = (x + 1)2 . (1)

Solución.

Primero determinemos la solución general yg de la ecuación homogénea

y 00 3y 0 + 2y = 0 (2)

Siendo la ecuación característica

m2 3m + 2 = 0

(m 2)(m 1) = 0,

63
siendo las raices

m1 = 1; m2 = 2

En consecuencia, la solución general de la ecuación (2) es

yg = C1 ex + C2 e2x

Ahora determinemos la solución particular yp de la ecuación no homogénea


(1). Buscamos de la forma:

yp = Ax2 + Bx + C,

siendo sus derivadas

yp0 = 2Ax + B

yp00 = 2A.

Sutituyendo yp y sus derivadas en la ecuación (1), se tiene:

2A 3(2Ax + B) + 2(Ax2 + Bx + C) = x2 + 2x + 1

2A 6Ax 3B + 2Ax2 + 2Bx + 2C = x2 + 2x + 1

2Ax2 + ( 6A + 2B)x + (2A 3B + 2C) = x2 + 2x + 1

Igualamos los coe…cientes de potencias iguales de x, obtenemos el sistema


8
< 2A = 1;
6A + 2B = 2;
:
2A 3B + 2C = 1:

Donde
1 5 15
A= ; B= ; C= .
2 2 4

Luego, la solución particular de la ecuación (1) es


1 2 5 15
yp = x + x+ .
2 2 4

64
Por tanto, la solución general de la ecuación (1) es
1 5 15
y = C1 ex + C2 e2x + x2 + x + .
2 2 4

Ejemplo.
Hallar la solución general de la ecuación

y 00 + y = cos x + sen x. (1)

Solución.
Comenzamos determinando la solución general yg de su correspondiente
ecuación homogenea

y 00 + y = 0 (2)

La ecuación caracteristica es

m2 + 1 = 0,

sus raices son complejas y conjugadas

m1 = i; m2 = i

De manera que la solución gerneral de la ecuación (2) es

yg = C1 cos x + C2 sen x

Seguidamente determinamos la solución particular yp de la ecuación (1).


Como R(x) = cos x+sen x es una solución de la ecuación homogénea. Buscamos
la solución particular de forma

yp = x(A cos x + B sen x)

yp = Ax cos x + Bx sen x

Sus derivadas son

65
yp0 = A cos x Ax sen x + Bx sen x + Bx cos x

yp0 = (Bx + A) cos x + ( Ax + B) sen x

yp00 = B cos x (Bx + A) sen x A sen x + ( Ax + B) cos x

yp00 = ( Ax + 2B) cos x (Bx + 2A) sen x

Sustituyendo yp y sus derivadas, tenemos:

( Ax + 2B) cos x (Bx + 2A) sen x + Ax cos x + Bx sen x = cos x + sen x

Ax cos x + 2B cos x Bx sen x 2A sen x + Ax cos x + Bx sen x = cos x + sen x

2B cos x 2A sen x = cos x + sen x

Igualamos los coe…cientes de cos x y sen x; obtenemos

2A = 1;
2B = 1:

Donde

1 1
A= , B= .
2 2

Reemplazando estos coe…cientes en la solución particular es


1 1
yp = x cos x + x sen x
2 2

Por tanto, la solución general de (1) es:


1 1
y = yg + yp = C1 cos x + C2 sen x x cos x + x sen x.
2 2

Ejemplo.

Hallar la solución general de la ecuación

y 00 y0 6y = 39 sen 3x. (1)

66
Solución.
Sea la ecuación homogénea

y 00 y0 6y = 0. (2)

La ecuación característica es

m2 m 6=0

(m 3)(m + 2) = 0

Las raices son

m1 = 3; m2 = 2

En consecuencia la solución general de la ecuación (2) es

yg = C1 e3x + C2 e 2x

Planteamos la solución particular de la forma

yp = A cos 3x + B sen 3x.

Y sus derivadas son

yp0 = 3A sen 3x + 3B cos 3x

yp00 = 9A cos 3x 9B sen 3x.

Sustituyendo la solución particular y sus derivadas en (1), tendremos

9A cos 3x 9B sen 3x ( 3A sen 3x + 3B cos 3x) 6(A cos 3x + B sen 3x) =

39 sen 3x
9A cos 3x 9B sen 3x+3A sen 3x 3B cos 3x 6A cos 3x 6B sen 3x = 39 sen 3x

15A cos 3x 15B sen 3x + 3A sen 3x 3B cos 3x = 39 sen 3x

( 15A 3B) cos 3x + (3A 15B) sen 3x = 39 sen 3x.

67
Igualando coe…cientes de cos 3x y sen 3x, obtenemos el sistema

15A 3B = 0;
3A 15B = 39:

15
=) 3B = 15A =) B = 3 A = 5A:

39
=) 3A 15( 5A) = 39, =) 3A + 75A = 39, =) A = 78 = 12 . .=) B = 5
2.

De manera que
1
A=
2
5
B= .
2

Reemplazando los coe…cientes en yp , La solución particular es:

1 5
yp = cos 3x sen 3x
2 2

Por tanto la solución general de (1) es:


1 5
y = yg + yp = C1 e3x + C2 e 2x
+ cos 3x sen 3x:
2 2

VARIACIÓN DE PARÁMETROS

Ahora desarrollaremos un método más general, llamado "variación de parámet-


ros", para determinar una solución particular de la ecuación

y 00 + P (x) y 0 + Q (x) y = R (x). (1)

Para emplear este método, es necesario conocer un conjunto fundamental de


soluciones de la ecuación homogénea

y 00 + P (x) y 0 + Q (x) y = 0, (2)

donde P (x), Q(x) y R(X) son funciones continuas en un intervalo.


Supongase que y1 y y2 son dos soluciones L.I. de la ecuación (2), entonces
la solución general de la ecuación homogénea (2) es

68
yg = C1 y1 (x) + C2 y2 (x). (3)

El método es análogo a la "reducción de orden", es decir, sustituimos las


constantes C1 y C2 por las funciones desconocidas v1 (x), v2 (x) e intentamos
hallar v1 y v2 tal que

yp = v1 y1 + v2 y2 , (4)

sea la solución particular de la ecuación (1). Para este propósito debemos


calcular las derivadas

yp0 = v1 y10 + v10 y1 + v2 y20 + v20 y2

yp0 = (v1 y10 + v2 y20 ) + (v10 y1 + v20 y2 ). (5)

Una segunda derivada complicaria más, entonces hacemos que el segundo


término entre parentesis se anule

v10 y1 + v20 y2 = 0. (6)

La ecuación (5) se reduce a

yp0 = v1 y10 + v2 y20 . (7)

Derivando una vez más, se tiene

yp00 = v1 y100 + v10 y10 + v2 y200 + v20 y20 . (8)

Sustituyendo las ecuaciones (4), (7) y (8) en (1), obtenemos

(v1 y100 + v10 y10 + v2 y200 + v20 y20 ) + P (v1 y10 + v2 y20 ) + Q (v1 y1 + v2 y2 ) = R (x).

Reagrupando los términos, resulta

v1 (y100 + P y10 + Qy1 ) + v2 (y200 + P y20 + Qy2 ) + v10 y10 + v20 y20 = R (x),

como y1 , y2 son soluciones de la ecuación (2), los dos términos entre paran-
tesis se anulan, para tener

69
v10 y10 + v20 y20 = R (x). (9)

Las ecuaciones (6) y (9) constituyen un sistema lineal de dos ecuaciones con
las incognitas v10 y v20 .

v10 y1 + v20 y2 = 0
v10 y10 + v20 y20 = R (x)

Resolviendo el sistema por la regla de Cramer, tenemos


y2 R (x) y1 R (x)
v10 = ; v20 = . (10)
W (y1 ; y2 ) W (y1 ; y2 )
Donde
y1 y2
W (y1 ; y2 ) = ,
y10 y20

es el Wronskiano de y1 y y2 , con W (y1 ; y2 ) 6= 0.


Integrando las ecuaciones de (10), tendremos
Z Z
y2 R (x) y1 R (x)
v1 = dx, v2 = dx.
W (y1 ; y2 ) W (y1 ; y2 )

Finalmente la solución particular de la ecuación (1) es


Z Z
y2 R (x) y1 R (x)
yp = y1 dx + y2 dx.
W (y1 ; y2 ) W (y1 ; y2 )

Por tanto la solución general de la ecuación (1) es

y = yg + yp .

Ejemplo.

Determinar la solución general de

4y 00 + 4y 0 + y = 3ex . (11)

Solución.

La ecuación (11) escribimos en la forma (1)´:


1 3
y 00 + y 0 + y = ex . (12)
4 4

70
primero, determinemos la solución general yg , de la ecuación homogénea

1
y 00 + y 0 + y = 0. (13)
4

Siendo la ecuación caracteristica


1
m2 + m + =0
4
p p
p p2 4q 1 1 1 1
m= = = .
2 2 2

Luego
m1 = m2 = 21 .
Como las raices son reales y repetidas, las soluciones son

y1 =e y2 = xe
1 1
2x , 2x .

Así, la solución general de la ecuación (13) es

yg = C1 e + C2 xe
1 1
2x 2x .

Ahora calculemos el Wronskiano de y1 , y2 .


1 1
y1 y 2 e 2x xe 2 x 1
W (y1 ; y2 ) = = 1 1x 1 =e x
xe x
+
y10 y20 e 2
1
e 2x xe
1
2x 2
2 2
1
xe x
=e x
.
2
De manera que,
Z Z 1 3 x Z 3 1x Z
xe 2x e xe 2
y2 R (x) 4 dx = 4 3
xe 2 x dx
3
v1 = dx = x
dx =
W (y1 ; y2 ) e e x 4

Integrando por partes, tenemos


u = x =) du = dx
Z
2 3
v = e 2 x dx = e 2 x .
3

71
Z Z
3 3 2 3x 2 1 3x 1 3x
xe 2 x dx = xe 2 + e 2 .
3 3
v1 = xe 2 e 2 x dx =
4 4 3 3 2 3

Para v2 , se tiene

Z Z e 3 x
1
Z
2xe
y1 R (x) 4 dx = 3 1 3x
v2
W (y1 ; y2 )
dx =
e x 4
e 3
2x =
2
e2 .

Luego, la solución particular es


1 3x 1 3x 1 3 1 x 1 x 1 x
xe 2 + e 2 e 2 x + e 2 x xe xe + e + xe =
1 1
yp = 2x =
2 3 2 2 3 2

1 x
3
e.
Por tanto, la solución general de la ecuación (11) es

y = yg + yp

1 x
y = C1 e + C2 xe e.
1 1
2x 2x +
3

Ejemplo.
Hallar la solución particular de la ecuación

y 00 + 4y = tan 2x (14)

Solución.

Determinemos la solución general de la ecuación homogenea

y 00 + 4y = 0. (15)

La ecuanción caracteristica es

m2 + 4 = 0
p
m= 4= 2i,

son raices complejas y conjugadas, donde =0y = 2. De manera que la

solución general de la ecuación homogénea (15) es

72
yg = C1 cos 2x + C2 sen 2x

Sea

y1 = cos 2x y y2 = sen 2x:

Ahora, calculamos el wronskiano de y1 y y2

y1 y2 cos 2x sen 2x
W (y1 ; y2 ) = = = 2 cos2 2x + 2 sen2 2x =
y10 y20 2 sen 2x 2 cos 2x

2 cos2 2x + sen2 2x = 2.

De manera que:
Z Z Z
y2 R (x) sen 2x tan 2x 1 sen 2x
v1 = dx = dx = sen 2x dx =
W (y1 ; y2 ) 2 2 cos 2x
Z
sen2 2x 1
dx
Z cosZ2x 2
2
1 1 cos 2x 1 1
= dx = cos 2x dx =
2 cos 2x 2 cos 2x
Z
1
(cos 2x sec 2x) dx
2
1 sen 2x ln jsec 2x + tan 2xj 1 1
= = sen 2x ln jsec 2x + tan 2xj.
2 2 2 4 4

Para v2 , tenemos
Z Z Z
y1 R (x) cos 2x tan 2x 1 sen 2x
v2 = dx = dx = cos 2x dx =
W (y1 ; y2 ) 2 2 cos 2x
Z
1 1
sen 2xdx = cos 2x.
2 4

Luego, la solución particular es

73
1 1 1
yp = sen 2x ln jsec 2x + tan 2xj cos 2x + cos 2x sen 2x
4 4 4

1 1 1
= sen 2x cos 2x cos 2x ln jsec 2x + tan 2xj sen 2x cos 2x
4 4 4
1
yp = cos 2x ln jsec 2x + tan 2xj.
4

TEMA No 4

SOLUCION EN SERIE DE POTENCIAS DE


ECUACIONES DIFERENCIALES

REPASO DE SERIE DE POTENCIAS

1. Una serie in…nita de la forma


1
X
an xn = a0 + a1 x + a2 x2 + a3 x3 + (1)
n=0

se llama una serie de potencias en x:

2. La serie.
1
X n 2
an (x x0 ) = a0 + a1 (x x0 ) + a2 (x x0 ) + (2)
n=0

es una serie de potencias en x x0 , que es algo más general que (1). Sin
embargo (2) se puede reducir a (1) sustituyendo x x0 por x:

3. Con relación a la disposición de sus puntos de convergencia, todas las


series de potencias caen en algunas de las tres categorias siguientes
1
X
1). n!xn = 1 + x + 2!x2 +
n=0

X1
xn x2
2). =1+x+ +
n=0
n! 2!

74
1
X
3). xn = 1 + x + x2 +
n=0
La primera serie es divergente para todo x 6= 0, la segunda es convergente
para todo x, la tercera es convergente para jxj < 1 y diverge para jxj > 1:

Radio de Convergencia.
Toda serie de potencia en x, tiene un radio de convergencia R, que tiene
la propiedad de que, la serie converge sí jxj < 1 y diverge sí jxj > 1, donde
0 6 R 6 1:

Para poder determinar el valor de R, consideremos. Sea una serie de con-


stantes no nula
1
X
un = u0 + u1 + u2 +
n=0

por cálculo elemental, sí el límite

un+1
lim = L,
n!0 un

existe, entonces por el "criterio del cociente" se asegura que:

Sí: L < 1, la serie converge.

Sí: L > 1, la serie diverge.

Para el caso de serie de potencias


1
X
an xn = a0 + a1 x + a2 x2 +
n=0

donde
an+1 xn+1 an+1
lim = jxj lim = L.
n!1 an xn n!1 an

Para la serie de potencias

75
1
X n 2
an (x x0 ) = a0 + a1 (x x0 ) + a2 (x x0 ) +
n=0

donde
n+1
an+1 (x x0 ) an+1
lim n = jx x0 j lim = L;
n!1 an (x x0 ) n!1 an

sí el límite existe, entonces.

Sí L < 1, la serie de potencias converge.

Sí L > 1, la serie de potencias diverge.

Sí L = 0, la prueba no es concluyente.

En concecuencia, la fórmula de radio de convergencia es

1 an
R= an+1 = n!0
lim
lim an+1
n!0 an

1
sí el limite existe. Como 0 6 L 6 1 entonces R = , si R = 0 si L = 1 y
L
R = 1 si L = 0.

Sí R es …nito y no nulo determina un "intervalo de convergencia" x0 R6

x 6 R + x0 , tal que la serie es convergente en su interior y divergente en el


exterior. Gra…camente tenemos

Una serie de potencias puede o no ser convergente en los puntos terminales


de su intervalo de convergencia.

Ejemplo.
Determinar radio e intervalo de convergencia de la serie de potencias.
X1 n
(x + 2)
n
.
n=1
3 n

76
Solución.

Primero determinamos radio de convergencia

1
3n n 3n+1 (n + 1) 3n 3 (n + 1)
R = lim = lim = lim =
n!1 1 n!1 n
3 n n!1 3n n
3n+1 (n + 1)

3 (n + 1)
lim =?
n!1 n

Aplicando algunos arti…cios matemáticos, para levantar la indeterminada,


tenemos
3 (n + 1) 1
3 1+
n n 1
R = lim n = lim = 3 lim 1 + n = 3 1 = 3.
n!1 n!1 1 n!1
n

1
NOTA: = 0.
1
Siendo el radio de convergencia R = 3. Ahora determinemos el intervalo de
convergencia, utilizando el criterio del cociente
n+1
(x + 2)
n+1
an+1 3n+1 (n + 1) 3n n (x + 2)
lim = lim n = lim n+1 n =
n!1 an n!1 (x + 2) n!1 3 (n + 1) (x + 2)
3n n
n (x + 2) n
lim = jx + 2j lim
n!1 3 (n + 1) n!1 3 (n + 1)
n
n 1 1
= jx + 2j lim = jx + 2j lim = jx + 2j < 1.
n!1 3 (n + 1) n!1 1 3
3 1+
n n

jx + 2j < 3

3 < x + 2 < 3.

De modo que la serie de potencias converge para 5 < x < 1. Ahora

analizaremos los puntos terminales del intervalo I de convergencia.

Para x = 5, la serie de potencias se convierte en la serie canónica alternada

77
X1 n X1 n X1 n
( 5 + 2) ( 3) ( 1)
= = ,
n=1
3n n n=1
3n n n=1
n

la cual es convergente.

Para x = 1, la serie se convierte en la serie armonica.


X1 n X1
(1 + 2) 1
nn
= ,
n=1
3 n=1
n

la cual es divergente. Por tanto, el intervalo de convergencia es

5 6 x < 1.

Ejemplo.
Determinar radio e intervalo de convergencia de la serie
1
X 4 n
(x 3)
n=1
n2 +2

Solución.
Primero determinamos radio de convergencia

4
2
an n2 +2 (n + 1) + 2
R = lim = lim = lim =
n!1 an+1 n!1 4 n!1 n2 + 2
2
(n + 1) + 2

n2 + 2n + 3 2 3
2
n + 2n + 3 1+ + 2
lim = lim n2 = lim n n = 1.
n!1 n2 + 2 n!1 n2 + 2 n!1 2
1+ 2
n2 n

Siendo el radio de convergencia R = 1. Ahora determinamos el intervalo de


convergencia, aplicando el criterio del cociente.
4 n+1
2 (x 3)
an+1 (n + 1) + 2
lim = lim =
n!1 an n!1 4 n
(x 3)
n2 + 2
n+1 n+1
4 (x n2 + 2
3) 4 (x 3) n2 + 2
lim h i = lim h i
n!1 2 n n!1 2 n
(n + 1) + 2 4 (x 3) (n + 1) + 2 4 (x 3)

78
(x 3) n2 + 2 n2 + 2
= lim = jx 3j lim =
n!1 n2 + 2n + 3 n!1 n2 + 2n + 3

n2 + 2 2
2
1+ 2
jx 3j lim n = jx 3j lim n
n!1 n2 + 2n + 3 n!1 2 3
1+ + 2
n 2 n n
1
= jx 3j = jx 3j 1 < 1.
1

De modo que la serie de potencias "Converge", como jx 3j < 1 es decir

1<x 3<1

1+3<x 3+3<1+3

2 < x < 4,

siendo el intervalo de convergencia: 2 < x < 4.

Analizaremos los puntos terminales del intervalo de convergencia.

Para x = 2, se tiene
X1 X1 X1 n X1 n
4 n 4 n 4 ( 1) 4 ( 1)
2
(x 3) = 2
(2 3) = =
n=1
n +2 n=1
n +2 n=1
n2 + 2 n=1
n2 + 2
La serie es convergente.

Para x = 4, se tiene
1
X 1
X X1 n X1
4 n 4 n 4 (1) 4
2+2
(x 3) = 2+2
(4 3) = 2+2
ó 2+2
n=1
n n=1
n n=1
n n=1
n

La serie es divergente. Por tanto, el intervalo de convergencia es: 2 6 x < 4.


4. Sea f (x) una función continua y que admite derivadas de todo orden
en jxj < R con R > 0 de f (x) se puede representar mediante una serie de
potencias de la forma
1
X f (n) (0) f 00 (0) 2
f (x) = xn = f (0) + f 0 (0) x + x +
n=0
n! 2!

donde
f (n) (0)
an = .
n!

79
Una forma de veri…car la validez de este desarrollo es mediante la fórmula
de Taylor

P
1 f (k) (0) n
f (x) = x + Rn (x).
k=0 k!

Donde Rn (x) es el resto

f (n+1) ( )
Rn (x) = ,
(n + 1)!

para 2 (0; 1).

Ejemplo.
Sean los dsarrollos alrededor del punto x0 , llamadas series Maclaurin
1 n
X x x2 x3
ex = =1+x+ + +
n=0
n! 2! 3!

1
X n x2n+1 x3 x5 x7
sen x = ( 1) =x + +
(2n + 1)! 3! 5! 7!
k=0

1
X n x2n x2 x4 x6
cos x = ( 1) =1 + +
n=0
(2n!) 2! 4! 6!

5. Una función que admite un desarrolla en serie de potencias de la forma


1
X n
f (x) = an (x x0 ) ,
n=0

en algún entorno del punto x0 , se dice que f (x) es "analitica" en x0 .

Ejemplo.
Las funciones ex , sen x y cos x son analíticas en x0 = 0.
SOLUCIÓN POR SERIE DE POTENCIAS DE ECUACIONES
DIFERENCIALES DE PRIMER ORDEN

Ejemplo.
Consideremos la ecuación diferencial

y0 + y = 0 (1)

Solución.
Suponemos que la solución es en forma de serie de potencias

80
y = a0 + a1 x + a2 x2 + a3 x3 + + an x + (2)

que convenge para jxj < R, con R > 0 y que tiene una solución análitica en
el origen, es decir, x0 = 0:

La ecuación (2) puede derivarse término a término.


y 0 = a1 + 2a2 x + 3a3 x2 + 4a4 x3 + + (n 1)
n 2
an 1x + nan xn 1
+ (n + 1)an+1 xn + (3)

Sustituyendo (2) y (3) en (1), tenemos


(a1 +2a2 x+3a3 x2 + +(n 1) an 1 xn 2 +nan xn 1
+(n+1)an+1 xn + )+

+ a0 + a1 x + a2 x2 + a3 x3 + + an 1x
n 1
+ an xn + an+1 xn+1 + = 0.

Agrupando los coe…cientes de potencia iguales de x, tenemos


(a1 + a0 ) + (2a2 + a1 ) x + (3a3 + a2 ) x2 + (4a4 + a3 ) x3 + + (nan +
n 1
an 1 )x + [(n + 1) an+1 + an ] xn + :::: = 0.
Igualando a cero los coe…ciente, resulta.
a1 + a0 = 0 ) a1 = a0
a1 a0
2a2 + a1 = 0 ) a2 = =
2 1 2
a2 a0
3a3 + a2 = 0 ) a3 = =
3 1 2 3
a3 a0
4a4 + a3 = 0 ) a4 = =
4 1 2 3 4
..
.
a0
nan + an 1 = 0 entonces an+1 = ( 1)n .
1 2 3 (n + 1)

Sustituyendo estos coe…cientes en la ecuación (2) obtenemos la solución de


(1) , en serie de potencias, esto es
a0 2 a0 3 a0 4 a0 n a0
y = a0 a0 x + x x + x + + x xn+1 +
2! 3! 4! n! (n + 1)!

x2 x3 x4 xn xn+1
y = a0 1 x+ + + + +
2! 3! 4! n! (n + 1)!

81
ó bien
y =

a0 e x .
Ejemplo.

Sea la ecuación diferencial

(1 + x) y 0 + y = 0 (1)

Determinar una solución en serie de potencias en el entorno x0 = 0.


Solución.

Suponemos que la solución es de la forma

y = a0 + a1 x + a2 x2 + a3 x3 + a4 x4 + (2)

Siendo las derivadas

y0 =

a1 +2a2 x+3a3 x2 +4a4 x3 + +(n 1)an 1 xn 2


+nan xn 1
+(n+1)an+1 xn +
(3)

Sustituyendo los desarrollos y y y 0 en (1), tenemos

(1 + x) a1 + 2a2 x + 3a3 x2 + 4a4 x3 + +

a0 + a1 x + a2 x2 + a3 x3 + a4 x4 + =0
(a1 + 2a2 x + 3a3 x + 4a4 x3 +
2
+ a1 x + 2a2 x2 + 3a3 x3 + 4a4 x4 + ) + (a0 +

a1 x + a2 x2 + a3 x3 + a4 x4 + ) = 0.

Agrupamos los términos semejantes

(a1 + a0 ) + (2a2 + 2a1 ) x + (3a3 + 3a2 ) x2 + (4a4 + 4a3 ) x3 + = 0.

82
Luego igualamos los coe…cientes
a1 + a0 = 0 ) a1 = a0

2a2 + 2a1 = 0 ) a2 = a1 = a0

3a3 + 3a2 = 0 ) a3 = a2 = a0

4a4 + 4a3 = 0 ) a4 = a3 = a0

..
.

Sustituyendo estos coe…cientes en (2) tenemos la solución

y = a0 1 x + x2 x3 + x4 +

ó simplemente

1
y = a0 .
1+x

Ejemplo.

Resolver la ecuación

2(x 1)y 0 3y = 0, (1)

por serie de potencias entorno al punto x0 = 0.


Solucion.

Suponemos que la solucion es de la forma

y = a0 + a1 x + a2 x2 + a3 x3 + a4 x4 + (2)

Su derivada es

y 0 = a1 + 2a2 x + 3a3 x2 + 4a4 x3 + + (n 1)an 1x


n 2
+ nan xn 1
+ (n +

1)an+1 xn + (3)

Sustituyendo (2) y (3) en (1), tenemos

83
2 (x 1) a1 + 2a2 x + 3a3 x2 + 4a4 x3 +
3 a0 + a1 x + a2 x2 + a3 x3 + a4 x4 + =0
2 3 4
2a1 x + 4a2 x + 6a3 x + 8a4 x + 2a1 4a2 x 6a3 x2 8a4 x3 +

3a0 3a1 x 3a2 x2 3a3 x3 3a4 x4 =0

Agrupamos los términos semejantes

( 2a1 3a0 ) + ( a1 4a2 ) x + (a2 6a3 ) x2 + (3a3 8a4 ) x3 +

(5a4 10a5 ) x4 + = 0.

Igualando términos se tiene;


3a0
2a1 3a0 = 0 ) a1 =
2
a1 3a0
a1 4a2 = 0 ) a2 = =
4 8
a2 3a0
a2 6a3 = 0 ) a3 = =
6 48
3a3 3a0
3a3 8a4 = 0 ) a4 = =
8 128

..
.
Siendo la solución
3 3 3 3 4
y = a0 1 x + x2 + x3 + x +
2 8 48 128

1 1
y = a0 = a0 .
2 (x 1) 2x 2

ECUACIONES DIFERENCIALES DE SEGUNDO ORDEN

Puntos Ordinarios.
Consideramos la ecuación lineal homogénea de segundo orden

y 00 + P(x) y 0 + Q(x) y = 0 (1)

84
donde P (x) y Q (x) son funciones continuas de x para nuestro análisis nos
restringiremos al caso en que P (x) y Q (x) deben ser análiticas en x0 , lo que
signi…ca que P (x) y Q (x) tiene un desarrollo en serie de potencias, válida en
algún entorno del punto x0 , en tal caso se dira que x0 es "punto ordinario" de
la ecuación (1), en caso contrario es un "punto singular".
Ejemplo.
Resolver la ecuación

y 00 + y = 0, (2)

en serie de potencias de x.
Solución.
Las funciones P (x) = 0, Q (x) = 1, ambas a la vez son analíticas en el punto
x0 = 0 (más en general para todo x), de manera que x0 = 0 es un punto
ordinario.
Buscamos la solución de la forma

y = a0 + a1 x + a2 x2 + a3 x3 + :::: + an 1x
n 1
+ an xn + an+1 xn+1 + ::: (3)

cuyas derivadas son

y 0 = a1 + 2a2 x + 3a3 x2 + 4a4 x3 + + (n 1) an 1x


n 2
+ nan xn 1
+ (n +

1)an+1 xn + (n + 2)an+2 xn+1 +


y 00 = 2a2 + 6a3 x + 12a4 x2 + + (n 2) (n 1) an 1x
n 3
+

(n 1) nan xn 2
+ n (n + 1) an+1 xn 1
+ (n + 1) (n + 2) an+2 xn +
(4)

Sustituyendo los desarrollos (3) y (4) en (2), tenemos


[2a2 + 6a3 x + 12a4 x2 + + (n 2) (n 1) an 1 xn 3 + (n 1) nan xn 2
+

n (n + 1) an+1 xn 1
+ (n + 1) (n + 2) an+2 xn + ]+

+ a0 + a1 x + a2 x2 + a3 x3 + a4 x4 + =0

Agrupamos los términos semejantes:

(2a2 + a0 ) + (6a3 + a1 ) x + (12a4 + a2 ) x2 + +

[(n 2) (n 1) an 1 + an 3 ] xn 3 + [(n 1) nan + an 2 ] xn 2

+ [n (n + 1) an+1 + an 1 ] xn 1 + [(n + 1) (n + 2) an+2 + an ] xn + =0

85
Igualando a cero cada coe…ciente, tenemos
a0
2a2 + a0 = 0 ) a2 =
1 2
a1
6a3 + a1 = 0 ) a3 =
1 2 3
a2 a0
12a4 + a2 = 0 ) a4 = =
3 4 1 2 3 4
a3 a1
20a5 + a3 = 0 ) a5 = =
4 5 1 2 3 4 5

Sustituyendo estos coe…cientes en (3), obtenemos la solución.


a0 2 a1 3 a0 4 a1 5
y = a0 + a1 x x x + x + x
2! 3! 4! 5!

x2 x4 x6 x3 x5 x7
y = a0 1 + + + a1 x + + ,
2! 4! 6! 3! 5! 7!

o simplemente

y = a0 cos x + a1 sen x

Ejemplo.

Resolver la ecuación

(1 + x) y 00 + y 0 = 0 (6)

en serie de potencias.

Solución.

Escribimos la ecuación en forma canónica


1
y 00 + y 0 = 0.
1+x
1
Como P (x) = y Q(x) = 0 son ambas son análiticas en el origen
1+x
x0 = 0, entonces x0 = 0 es un punto ordinario.
Suponemos que la solución es de forma siguiente

y = a0 + a1 x + a2 x2 + a3 x3 + a4 x4 + (7)

o bien

86
1
X
y= an xn
n=0

Las derivadas son


1
X
y0 = nan xn 1

n=0

1
X
y 00 = (n 1) nan xn 2

n=0

Sea n n + 1 para y 0 ; entonces:


1
X
y0 = (n + 1) an+1 xn
n=0

Sea n n + 2 para y 00 ;entonces


1
X
y 00 = (n + 1) (n + 2) an+2 xn
n=0

Sustituyendo los desarrollos y 0 y y 00 en (6) , tenemos


1
X 1
X
(1 + x) (n + 1) (n + 2) an+2 xn + (n + 1) an+1 xn = 0
n=0 n=0
1
X 1
X
(n + 1) (n + 2) an+2 xn + (n + 1) (n + 2) an+2 xn+1 +
n=0 n=0

1
X
(n + 1) an+1 xn = 0
n=0

Para la segunda sumatoria sea: n n 1


1
X 1
X 1
X
(n + 1) (n + 2) an+2 xn + n (n + 1) an+1 xn + (n + 1) an+1 xn = 0
n=0 n=1 n=0

1
X 1
X
1 2a2 + (n + 1) (n + 2) an+2 xn + n (n + 1) an+1 xn + a1 +
n=1 n=1
1
X
(n + 1) an+1 xn = 0
n=1

87
Agrupamos los términos semejantes, se tiene
1
X
(1 2a2 + a1 )+ [(n + 1) (n + 2) an+2 + n (n + 1) an+1 + (n + 1) an+1 ] xn = 0
n=1

1 h
X i
2
(1 2a2 + a1 ) + (n + 1) (n + 2) an+2 + n (n + 1) an+1 xn = 0
n=1

Igualamos a cero los coe…cientes,resulta


a1
1 2a2 + a1 = 0 ) a2 =
1 2

2
(n + 1) (n + 2) an+2 + n (n + 1) an+1 = 0

siendo
2
n (n + 1) an+1 n (n + 1) an+1 n (n + 1)
an+2 = = = an+1 , para
(n + 1) (n + 2) (n + 2) (n + 2)

n : 1; 2; 3; : : :

Está ecuación es conocida como la "formula de recurrencia".


a1
a2 =
1 2

2 2 a1 a1
a3 = a2 = =
3 3 1 2 3

3 3 a1 a1
a4 = a3 = =
2 2 3 2

12 12 a1 6
a5 = a4 = = a1
5 5 2 5

10 10 6
a6 = a5 = a1 = 4a1
3 3 5

..
.

Sustituyendo estos coe…cientes , en la ecuación (7) ,obtenemos la solución


a1 2 a1 3 a1 4 6 5
y = a0 + a1 x x + x x + x 4a1 x6 +
2 3 2 5

88
Ejemplo.

Resolver por serie de potencias, la ecuación

y 00 (x + 1) y = 0 (8)

Solución.
Como P (x) = 0 y Q (x) = (x + 1) como las funciones, ambas son análiticas
en x0 = 0. Suponemos que la ecuación tiene la solución de la forma

y = a0 + a1 x + a2 x2 + a3 x3 + a4 x4 + (9)

o bien
1
X 1
X
y= an xn , con y0 = (n + 1) an+1 xn , y y 00 =
n=0 n=0

1
X
(n + 1) (n + 2) an+2 xn .
n=0

Sustituyendo los desarrollos en y0 y y 00 en (8), tenemos


1
X 1
X
(n + 1) (n + 2) an+2 xn (x + 1) an xn = 0
n=0 n=0

1
X 1
X 1
X
(n + 1) (n + 2) an+2 xn an xn+1 an xn = 0
n=0 n=0 n=0

Debemos igualar las potencias de x de la segunda sumatoria, para ello efec-


tuamos una operación de asignación a n n 1, tenemos

1
X 1
X 1
X
(n + 1) (n + 2) an+2 xn an 1x
n 1+1
an xn = 0
n=0 n 1=0 n=0

1
X 1
X 1
X
(n + 1) (n + 2) an+2 xn an 1x
n
an xn = 0
n=0 n=1 n=0

Desarrollamos un primer término de la primera y tercera sumatoria, respec-


tivamente. Entonces, sí n = 0, tenemos

89
1
X 1
X 1
X
1 2a2 + (n + 1) (n + 2) an+2 xn an 1x
n
a0 an xn = 0
n=1 n=1 n=1

Agrupando los términos semejantes, tenemos


"1 1 1
#
X X X
(1 2a2 a0 ) + (n + 1) (n + 2) an+2 an 1 an xn = 0
n=1 n=1 n=1

1
X
(1 2a2 a0 ) + [(n + 1) (n + 2) an+2 an 1 an ] xn = 0
n=1

Igualando a cero los coe…entes,se tiene


a0
1 2a2 a0 = 0 ) a2 =
1 2

Luego, la formula de recurrencia es:


(n + 1) (n + 2) an+2 an 1 an = 0

(n + 1) (n + 2) an+2 = an 1 + an

an 1 + an
an+2 =
(n + 1) (n + 2)

Para n = 1; 2; 3; 4; : : :

Tenemos los siguientes coe…cientes


a0 + a1 a0 a1
a3 = = +
2 3 2 3 2 3
a0
a1 + a2 a1 a2 a1 a1 a0
a4 = = + = + 1 2 = +
3 4 3 4 3 4 3 4 3 4 3 4 1 2 3 4
a0 a0 a1
a2 + a3 a2 a3 + a0 a0 + a1
a5 = = + = 1 2 + 2 3 2 3 = +
4 5 4 5 4 5 4 5 4 5 1 2 4 5 2 3 4 5
a0 a0 a1 3a0 + a0 a1 4a0 a1
= + + = + = +
1 2 4 5 2 3 4 5 2 3 4 5 1 2 3 4 5 5! 5! 5!

..
.

Sustituyendo estos coe…cientes en al aecuación (9), la solución es:

90
a0 2 a0 + a1 3 2a1 + a0 4 4a0 + a1 5
y = a0 + a1 x + x + x + x + x +
2! 3! 4! 5!

Puntos Singulares.
Consideremos la ecuación diferencial líneal de segundo orden

y 00 + P (x) y 0 + Q (x) y = 0 (1)

El punto x0 , es un "punto singular" de la ecuación (1), si una de las


funciones P (x), Q (x)o ambas a la vez no son analíticas en x0 , en caso contrario
es un "punto ordinario" .

Ejemplo.
Sea la ecuación

x2 y 00 + 3xy 0 2y = 0

Determinamos los punto singulares y ordinarios.


Solución.

Primero escribimos la ecuación en la siguiente forma


3 0 2
y 00 + y y=0
x x2
3 2
Donde: P (x) = y Q (x) = , de manera que el punto singular es
x x2
x0 = 0. Esto signi…ca que P (x) como Q (x) no son análiticas en el punto x = 0,
y los puntos ordinarios son todos los valores de x, excepto x0 = 0:

Ejemplo.
Determinar los puntos singulares y ordinarios de la ecuacion de "Bessel"

x2 y 00 + xy 0 + x2 n2 y = 0

donde n es una constante.

Solución.
Escribimos la ecuacion de forma
1 0 x2 n2
y 00 + y + y=0
x x2

1 0 n2
y 00 + y + 1 y=0
x x2

91
1 n2
Donde P (x) = , Q (x) = 1 . De manera que x = 0 es un punto
x x2
singular de la ecuacion de Bessel. Los puntos ordinarios son todos los valores
de x, excepto el cero.

Ejemplo
consideramos la ecuación de "Legendre"

1 + x2 y 00 2xy 0 + a (a + 1) y = 0

donde a es una constante. Determinar los puntos singulares y regulares.


Solución.
La ecuación escribimos en su forma más simple.

2x 0 a (a + 1)
y 00 y + y=0
1 x2 1 x2

2x a (a + 1)
Donde: P (x) = 2
, Q (x) = . Para determinar los puntos
1 x 1 x2

singulares de la ecuación de "Legendre", resolvemos la ecuación

1 x2 = 0 ) x = 1

De manera que los puntos singulares son: x = 1. Los puntos ordinarios


son todos los valores de x , excepto 1.

Puntos Singulares Regulares.

De…nición.
Se dice que el punto singular x0 , de la ecuación (1) , es un "punto singular
2
regular " si tanto (x x0 ) P (x) como (x x0 ) Q (x) son analíticas en x0 , en
caso contrario se llama punto singular irregular.

Ejemplo.
Consideremos la ecuación de "Bessel" en su forma estándar es de la siguiente
manera
1 0 x2 n2
y 00 + y + y = 0:
x x2

92
Clasi…car los puntos singulares.

Solución.
El punto singular de la ecuación "Bessel" es x0 = 0. Luego, hacemos la
prueba
1
(x x0 ) P(x) = (x 0) =1
x

2 2 x2 n2
(x x0 ) Q(x) = (x 0) = x2 n2 :
x2

Esto muestra que x0 = 0 es un punto singular regular.

Ejemplo.
Ahora consideremos la ecuación de "Legendre".

2x 0 a (a + 1)
y 00 y + y = 0:
1 x2 1 x2
Clasi…car los puntos singulares.

Solución.

Los punto singulares de la ecuación de Legendre son x0 = 1.

2x a (a + 1)
Donde: P (x) = y Q (x) = .
1 x2 1 x2

Hacemos la prueba para el punto singular x0 = 1.


2x 2x (x 1) 2x (x 1)
(x x0 ) P (x) = (x 1) = = =
1 x2 (x2 1) (1 + x) (x 1)

2x
1+x
2
2 2 a (a + 1) a (a + 1) (x 1) a (a + 1) (x 1)
(x x0 ) Q (x) = (x 1) 2
= =
1 x (1 + x) (x 1) x+1

De manera que x0 = 1, es un punto singular regular. En forma análoga se


veri…ca que el punto singular x0 = 1 es un punto singular regular.

Ejemplo.
Clasi…car los puntos singulares de la siguiente ecuación
2
x2 1 y 00 + (x + 1) y 0 y=0

Solución.
Primero la ecuación escribimos en forma canónica

93
(x + 1) 1
y 00 + 2y
0
2y =0
(x2 1) (x2 1)

o simplemente
1 1
y 00 + 2y
0
2 2y = 0.
(x + 1) (x 1) (x + 1) (x 1)

1 1
Donde P (x) = 2 y Q(x) = 2 2
(x + 1) (x 1) (x + 1) (x 1)
Para obtener los puntos singulares, resolvemos la ecuacion.
2 2
(x + 1) (x 1) = 0:

De manera que los puntos singulares son x0 = 1.

Ahora veri…quemos para x0 = 1


1 1
(x x0 ) P (x) = (x 1) 2 =
(x + 1) (x 1) (x + 1) (x 1)

Para: x0 = 1, resulta
1 1 1
) = = =1
(x + 1) (x 1) 0 2 0

Esto muestra que la función no esta de…nida y que x0 = 1, es un punto


singular irregular.
Para x0 = 1.
1 1
(x x0 ) P (x) = (x + 1) 2 = 2
(x + 1) (x 1) (x 1)
" #
2 2 1 1
(x x0 ) Q(x) = (x + 1) 2 2 = 2
(x + 1) (x 1) (x 1)
1 1 1 1
) 2 = = =
(x 1) x2 2x + 1 1+2+1 4

Esto muestra que x0 = 1; es un punto singular regular.


Ejemplo.

Clasi…car los puntos singulares de la ecuación

x3 (1 x) y 00 + (3x + 2) y 0 + xy = 0

94
Solución.

Escribimos la ecuación en la forma siguiente


(3x + 2) 0 x
y 00 + 3
y + 3 y=0
x (1 x) x (1 x)

(3x + 2) 0 1
y 00 + y + 2 y=0
x3 (1 x) x (1 x)

3x + 2 1
Como: P (x) = 3 4
y Q (x) = 2
x x x (1 x)

Para obtener los puntos singulares , resolvemos la ecuación:


x3 (1 x) = 0

x3 = 0 y 1 x=0

x=0 y x=1

De manera que los puntos singulares son x0 = 0 y x0 = 1.

Veri…camos para x0 = 0
3x + 2 x (3x + 2) 3x + 2
(x x0 ) P (x) = (x 0) = 3 = 2
x3 (1 x) x (1 x) x (1 + x)

Para: x0 = 0;resulta
3x + 2 0+2 2
= = =1
x2 (1 + x) 0 0

Esto muestra que x0 = 0, es un punto singular irregular.

Veamos para x0 = 1, tenemos


3x + 2 3x + 2
(x x0 )P(x) = (x 1) 3 =
x (1 x) x3

2 1 2 1 x 1
(x x0 )2 Q(x) = (x 1) = (x 1) =
x2 (1 x) x2 (x 1) x2

Como: x0 = 1
x 1 1 1 0
2
= = =0
x 1 1
Esto muestra que x0 = 1, es un punto singular regular.

METODO DE FROBENIUS
Si x = x0 es un punto singular regular de la ecuación

95
y 00 + P (x)y 0 + Q(x)y = 0 (1)

entonces, existe al menos una solución en serie de Frobenius de la forma:

1
X 1
X
y = (x x0 )r an (x x0 )n = an (x x0 )n+r
n=0 n=0

donde r es un número constante, que debe determinarse. Estudiaremos


solamente cuando el punto singular regular sea x0 = 0; de manera que
1
X 1
X
r n
y=x an x = an xn+r
n=0 n=0

La ecuación

r(r 1) + p0 r + q0 = 0,

se llama ecuación indicial de la ecuación (1), cuyas raices son r1 y r2 con


r1 > r2 siendo

p0 = lim (x x0 )P (x) y q0 = lim (x x0 )2 Q(x).


x!x0 x!x0

Teorema.
La ecuación (1), tiene dos soluciones Linealmente Independientes (L.I.) y1
y y2 si se dan los siguientes casos:
CASO I. Si r1 r2 no es un entero, entonces
1X
y1 = xr1 an xn ; a0 6= 0
n=0
1
X
r2
y2 = x bn xn ; b0 6= 0.
n=0

CASO II. Si r1 = r2 , son raices iguales, entonces.


1X
y1 = xr1 an xn ; a0 6= 0
n=0
1X
y2 = y1 ln jxj + xr2 bn xn ; b0 6= 0.
n=0

96
CASO III. Si r1 r2 = N; donde N es un número entero positivo, entonces
1
X
y1 = an xn+r1 ; a0 6= 0
n=0

1
X
y2 = Cy1 ln x + bn xn+r2 ,
n=0

donde C es una constante que puede ser cero.

0.1 CASO I.

Ejemplo.
Encuentre dos soluciones linealmente independientes usando el método de
Frobenios alrededor
del punto x0 = 0, para la ecuación
2xy 00 + (1 + x)y 0 2y = 0.

Solución.
La ecuación escribimos en la forma
1+x 0 1
y 00 + y y=0
2x x
1+x 1
Donde P (x) = y Q(x) = , vemos que x0 = 0, evidentemente es un
2x x
punto singular regular, siendo

1+x x(1 + x) 1+x 1


p0 = lim (x x0 )P (x) = lim x = lim = lim = ,
x!x0 x!0 2x x!0 2x x!0 2x 2

1 x2
q0 = lim (x x0 )2 Q(x) = lim x2 = lim = lim x = 0.
x!x0 x!0 x x!0 x x!0

De manera que la ecuación inicial es:


1 1 1
r(r 1) + p0 r + q0 = r(r 1) + r + 0 = r r 1+ 2 =r r 2 = 0.
2

Igualando cada factor a cero las raices indiciales son


r2 = 0
1 1
r1 = 0 ) r1 = .
2 2
1 1
Como r1 r2 = 0 = , la diferencia de las raices "no es un entero
2 2
positivo", entonces las soluciones son de la forma:

97
1
X 1
X
y = xr an xn = an xn+r
n=0 n=0

cuyas derivadas son:


1
X
y0 = (n + r)an xn+r 1

n=0

1
X
y 00 = (n + r 1)(n + r)an xn+r 2

n=0

Sustituyendo y y sus derivadas, en la ecuación inicial, se tiene.


2xy 00 + (1 + x)y 0 2y = 0
1
X 1
X 1
X
2x (n+r 1)(n+r)an xn+r 2
+(1+x) (n+r)an xn+r 1
2 an xn+r = 0
n=0 n=0 n=0

1
X 1
X 1
X
2 (n + r 1)(n + r)an xn+r 1
+ (n + r)an xn+r 1
+ (n + r)an xn+r
n=0 n=0 n=0

1
X
2 an xn+r = 0
n=0
1
X 1
X
(n + r) [2(n + r 1) + 1] an xn+r 1
+ [(n + r) 2] an xn+r = 0
n=0 n=0

1
X 1
X
(n + r)(2n + 2r 1)an xn+r 1
+ (n + r 2)an xn+r = 0
n=0 n=0

Ajustamos el índice de la primera sumatoria sustituyendo n n + 1,


1
X 1
X
(n + r + 1) [2(n + 1) + 2r 1] an+1 xn+r + (n + r 2)an xn+r = 0
n= 1 n=0

1
X 1
X
(n + r + 1)(2n + 2 + 2r 1)an+1 xn+r + (n + r 2)an xn+r = 0
n= 1 n=0

1
X 1
X
(n + r + 1)(2n + 2r + 1)an+1 xn+r + (n + r 2)an xn+r = 0.
n= 1 n=0

98
Desarrollamos un término para n = 1 en la primera serie, esto es
1
X 1
X
r 1 n+r
r(2r 2+1)a0 x + (n+r+1)(2n+2r+1)an+1 x + (n+r 2)an xn+r = 0
n=0 n=0

Agrupamos los coe…cientes de potencias iguales de x,


1
X
r(2r 1)a0 xr 1
+ [(n + r + 1)(2n + 2r + 1)an+1 + (n + r 2)an ] xn+r = 0:
n=0

(2)

Igualando a cero los coe…cientes de potencias iguales de x, resulta


r(2r 1)a0 = 0,

(n + r + 1)(2n + 2r + 1)an+1 + (n + r 2)an = 0. (3)

Como a0 6= 0 en (2) entonces r(2r 1) = 0 que es la ecuación inicial. Luego

de (3), obtenemos la fórmula de recurrencia

(n + r 2)an
an+1 = ; n = 0; 1; 2; : : : (4)
(n + r + 1)(2n + 2r + 1)

1
Para r = r1 = la ecuación (4) se reduce a
2
1
n+ 2 an
2 (2n 3)an
an+1 = = ; n = 0; 1; 2; : : :
1 1 2(n + 1)(2n + 3)
n+ +1 2n + 2 +1
2 2

Sí hacemos en esta ecuación n = 0; 1; 2; : : :, nos da


(2 0 3)a0 3 3
a1 = = a0 = a0 ,
2(0 + 1)(2 0 + 3) 2 3 2 3
(2 1 3)a1 1 1 3 3
a2 = = a1 = a0 = a0 ,
2(1 + 1)(2 1 + 3) 2 2 5 4 5 2 3 2 3 4 5

(2 2 3)a2 4 3 1 3
a3 = = a2 = a0 =
2(2 + 1)(2 2 + 3) 2 3 (4 + 3) 6 7 2 3 4 5

3
a0 ,
2 3 4 5 6 7

99
(2 3 3)a3 3 3 32
a4 = = a0 = a0 ,
2(3 + 1)(2 3 + 3) 8 9 2 3 4 5 6 7 2 3 4 5 6 7 8 9

..
.
En consecuencia la primera solución linealmente independiente de la ecuación
inicial en serie de Frobenius es:
1
X
r 1 3 3 3 32
y1 = x an xn = x 2 a0 + a0 x + a0 x2 a0 x3 + a0 x4 + :::::
n=0
3! 5! 7! 9!

1 3 3 3 3 32 4
y1 = a0 x 2 1+ x + x2 x + x + ::::: .
3! 5! 7! 9!

Para r = r1 = 0; la ecuación (4), se reduce a


(n 2)an
an+1 = ; n = 0; 1; 2; : : :
(n + 1)(2n + 1)

(0 2)a0 2
a1 = = a0 = 2a0 ,
(0 + 1)(2 0 + 1) 1 1

a1 1 1
a2 = = 2a0 = a0 ,
2 3 2 3 3
a2
a3 = = 0,
3 5
a3
a4 = = 0,
4 7
..
.
En consecuencia, la segunda solución linealmente independiente es:
1
X 1 x2
y2 = xr an xn = x0 a0 + 2a0 x + a0 x2 + 0 + : : : = a0 1 + 2x + :
n=0
3 3

Por tanto, la solución general es:

y = C1 y1 (x) + C2 y2 (x).

1 3 3 3 3 3 x2
y = C1 x 2 1+ x + x2 x + x4 + + C2 1 + 2x + :
3! 5! 7! 9! 3

100
Finalmente, las dos soluciones L.I. de la ecuación inicial en serie de Frobenius
son:
1 3 3 3 3 3
y1 = x 2 1+ x + x2 x + x4 + y y2 =
3! 5! 7! 9!
x2
a0 1 + 2x + .
3

CASO III
a) Caso No Logarítmico
Ejemplo.

Hallar dos soluciones liealmente independientes usando el método de Frobe-


nius al rededor del punto
x0 = 0, para la ecuación
x2 y 00 2xy 0 + (x2 + 2)y = 0. (1)
Solución
La ecuacion escribimos de la forma:

2 0 x2 + 2
y y + y =0
x x2
2 x2 + 2
Donde P (x) = y Q(x) = .
x x2
2 2
Como xP (x) = 2 y x Q(x) = x + 2 , son analíticas en el origen entonces
el punto x0 = 0 es un punto singular regular, siendo p0 = 2 y q0 = 2: De
manera que la ecuación inicial es:

r(r 1) + p0 r + q0 = r(r 1) 2r + 2 = r2 r 2r + 2 = r2 3r + 2 =
(r 2)(r 1) = 0.

Las raíces iniciales son: r1 = 2 y r2 = 1. Como: r1 r2 = 2 1 = 1 la


diferencia de las raices es un entero positivo, entonces una solución linealmente
independiente en serie de Frobenius es:

P
1 P
1
y = xr an xn = an xn+r (2)
n=0 n=0
Sus derivadas son:
P
1 P
1
y0 = (n + r)an xn+r 1
, y 00 = (n + r)(n + r 1)an xn+r 2
.
n=0 n=0

Sustituimos (2) y sus derivadas en (1):

P
1 P
1 P
1
x2 (n+r)(n+r 1)an xn+r 2
2x (n+r)an xn+r 1
+(x2 +2) an xn+r =
n=0 n=0 n=0
0

101
P
1 P
1 P
1 P
1
(n+r)(n+r 1)an xn+r 2 (n+r)an xn+r + an xn+r+2 +2 an xn+r =
n=0 n=0 n=0 n=0
0

Ajustamos el índice de la tercera sumatoria, reemplazando n ! n 2,


P
1 P1 P1 P
1
(n+r)(n+r 1)an xn+r 2 (n+r)an xn+r + an 2 xn+r +2 an xn+r =
n=0 n=0 n=2 n=0
0

Desarrollamos los términos para n = 0; 1, tenemos


P
1
r(r 1)a0 xr + (r + 1)ra1 xr+1 + (n + r)(n + r 1)an xn+r 2ra0 xr 2(r +
n=2
P
1 P1
1)a1 xr+1 2 (n + r)an xn+r + an 2x
n+r
n=2 n=2
P
1
+2a0 xr + 2a1 xr+1 + 2 an xn+r = 0
n=2
Agrupamos los coe…cientes de potencias iguales de x,
P
1
[r(r 1) 2r + 2]a0 xr + [(r + 1)r 2(r + 1) + 2]a1 xr+1 + [(n + r)(n +
n=2
r 1)an 2(n + r)an + an 2 + 2an ]xn+r = 0

Opeaciones Auxiliares
(n+r)(n+r 1)an 2(n+r)an +2an = [(n+r)2 (n+r) 2(n+r)+2]an =
[(n + r)2 3(n + r) + 2]an
= [(n + r) 2][(n + r) 1]an = [n + r 2][n + r 1]an

Luego
P
1
(r 2)(r 1)a0 xr +r(r 1)a1 xr+1 + [(n+r 2)(n+r 1)an +an 2 ]x
n+r
=0
n=2
Igualando a cero los coe…cientes de potencias iguales de x, tendremos
(r 2)(r 1)a0 = 0
r(r 1)a1 = 0 (3)
(n + r 2)(n + r 1)an + an 2 = 0 para n 2.

Como: a0 6= 0 (Por Hipótesis), entonces tenemos la ecuación indicial que es:

(r 2)(r 1) = 0
Ahora pasamos a analizar la ecuación (3) evaluando las raices indiciales en
la ecuación (3), es decir,
Para r = r1 = 2, en la ecuacion (3) resulta
2(2 1)a1 = 2 1a1 = 2 a1 = 0, entonces a1 = 0.

Para r = r2 = 1; en la ecuacion (3) resulta


1(1 1)a1 = 1 0a1 = 0 a1 = 0, entonces a1 6= 0. Esto signi…ca que a1 es
arbitrario.

102
Con a1 6= 0, y n 2, obtenemos la fórmula de recurencia.

1
an = an 2 para n 2. (4)
(n + r 2)(n + r 1)
Para determinar dos soluciones linealmente independientes de la ecuación
(1) para el caso r1 = r2 + N , donde N es un entero positivo, se recomienda
tomar la raíz menor.
Para r = r2 = 1, la ecuación (4), se reduce a
1
an = an 2 para n 2. (5)
(n 1)n

Hacemos n = 2; 3; 4; : : : y a1 6= 0 en (5)

1
a2 = 1 2 a0 ,
1
a3 = 2 3 a1 ,
1 1 1 1
a4 = 3 4 a2 = 34 1 2 a0 = 1 2 3 4 a0 ,
1 1 1 1
a5 = 4 5 a3 = 45 2 3 a1 = 2 3 4 5 a1 ,
..
.

Por tanto, la solución general es:

y = xr (a0 + a1 x + a2 x2 + a3 x3 + : : :)
2 4 3 5
y = a0 x 1 x2! + x4! : : : + a1 x x x3! + x5! ::: .

Esta solución generales es de la forma y = C1 y1 (x) + C2 y2 (x).

Finalmente, las dos soluciones L.I. de la ecuación (1) en serie de Frobenius


son:

P
1 x2n P
1 x2n+1
y1 = x ( 1)n , y2 = x ( 1)n .
n=0 (2n)! n=0 (2n + 1)!

Ejemplo

Hallar dos soluciones liealmente independientes usando el método de Frobe-


nius al rededor del punto
x0 = 0, para la ecuación
x2 (1 + 2x)y 00 + 2x(1 + 6x)y 0 2y = 0 (6)
Solución
La ecuacion escribimos de la forma:

2(1 + 6x) 0 2
y 00 + y y = 0.
x(1 + 2x) x2 (1 + 2x)

103
2(1 + 6x) 2
Donde P (x) = y Q(x) = .
x(1 + 2x) x2 (1 + 2x)
2(1 + 6x) 2
Como xP (x) = y x2 Q(x) = , son analíticas en el
(1 + 2x) (1 + 2x)
origen entonces el punto x0 = 0 es un punto singular regular, siendo p0 = 2 y
q0 = 2: De manera que la ecuación inicial es:
r(r 1) + p0 r + q0 = r(r 1) + 2r 2 = r2 r + 2r 2 = r2 + r 2 =
(r 1)(r + 2) = 0.

Las raíces iniciales son: r1 = 1 y r2 = 2. Como: r1 r2 = 1 ( 2) = 3 la


diferencia de las raices es un entero positivo, entonces una solución linealmente
independiente en serie de Frobenius es:
P
1 P
1
y = xr an xn = an xn+r (7)
n=0 n=0

Sus derivadas son:


P
1 P
1
y0 = (n + r)an xn+r 1
, y 00 = (n + r)(n + r 1)an xn+r 2
.
n=0 n=0
Sustituimos (7) y sus derivadas en (6):
P
1 P
1
x2 (1 + 2x) (n + r)(n + r 1)an xn+r 2 + 2x(1 + 6x) (n + r)an xn+r 1
n=0 n=0
P
1
2 an xn+r = 0
n=0
P
1 P
1 P
1
(n + r)(n + r 1)an xn+r + 2 (n + r)(n + r 1)an xn+r+1 + 2 (n +
n=0 n=0 n=0
P
1 P
1
r)an xn+r + 12 (n + r)an xn+r+1 2 an xn+r = 0
n=0 n=0

Ajustamos el índice de la segunda y la cuarta sumatoria, reemplazando n !


n 1,

P
1 P
1
(n + r)(n + r 1)an xn+r + 2 (n + r 1)(n + r 2)an 1x
n+r
n=0 n=1
P
1 P
1 P
1
+2 (n + r)an xn+r + 12 (n + r 1)an 1x
n+r
2 an xn+r = 0
n=0 n=1 n=0

Desarrollamos los primeros términos para n = 0, tenemos


P
1 P
1
r(r 1)a0 xr + (n + r)(n + r 1)an xn+r + 2 (n + r 1)(n + r
n=1 n=1
n+r
P
1
2)an 1x + 2ra0 xr + 2 (n + r)an xn+r
n=1
P
1
n+r
P
1
+12 (n + r 1)an 1x 2a0 xr 2 an xn+r = 0.
n=1 n=1
Agrupamos los coe…cientes de potencias iguales de x.
P
1
[r(r 1)+2r 2]a0 xr + [(n+r)(n+r 1)an +2(n+r 1)(n+r 2)an 1+
n=1

104
2(n + r)an + 12(n + r 1)an 1 2an ]xn+r = 0
Opeaciones Auxiliares
(n+r)(n+r 1)an +2(n+r)an 2an = [(n+r)2 (n+r)+2(n+r) 2]an =
[(n + r)2 + (n + r) 2]an =
(n + r 1)(n + r + 2)an .
Por otra parte
2(n+r 1)(n+r 2)an 1 +12(n+r 1)an 1 = 2(n+r 1)(n+r 2+6)an 1 =
2(n + r 1)(n + r + 4)an 1 .
Luego:
P
1
(r 1)(r + 2)a0 xr + [(n + r 1)(n + r + 2)an + 2(n + r 1)(n + r +
n=1
4)an 1 ]xn+r = 0.
Igualando a cero los coe…cientes de potencias iguales de x, tendremos
(r 1)(r + 2)a0 = 0,
(n + r 1)(n + r + 2)an + 2(n + r 1)(n + r + 4)an 1 = 0 para n 1.
(8)

Como a0 6= 0 (por Hipótesis), entonces tenemos la ecuación indicial que es:


(r 1)(r + 2) = 0.
Para determinar dos soluciones linealmente independientes de la ecuación
(6), para el caso el caso r1 = r2 + N , donde N es un entero positivo, se
recomienda tomar la raíz menor r = r2 = 2, con a0 6= 0 y analizamos
con cuidado el comportamiento de la ecuación de recurrencia (8). Así que, la
ecuación (8) se reduce a

(n 3)nan + 2(n 3)(n + 2)an 1 = 0, n 1. (9)


Para n 6= 3 despejamos an y obtenemos la fórmula de recurrencia

2(n + 2)
an = an 1 para n 1 y n 6= 3. (10)
n
Hacemos n = 1, 2 en (9)

a1 = 2 3a0 ,
a2 = 224 a1 = 4( 2 3a0 ) = 2 3 4a0 .
Para n = 3, la ecuación (9) toma la forma 0 a3 + 0 a2 = 0, vemos que a3
no puede obtenerse hasta que n > 3. Entonces a3 elegimos como la segunda
constante arbitraria para obtener los demás coe…cientes. Hacemos n = 4; 5; 6; : : :

en (10)
a4 = 246 a3 = 3a3 ,
a5 = 257 a4 = 257 ( 3a3 ) = 2 35 7 a3 ,
a5 = 268 a4 = 83 2 35 7 a3 = 2 57 8 a3 ,
..
.
Por tanto, la solución general es:

105
2 42 5 112 6
y = a0 x 1 6x + 24x2 + a3 x 2
x3 3x4 + 5 x 5 x + :::

Finalmente, las dos soluciones L.I. de la ecuación (1) en serie de Frobenius


son:

2 42 5 112 6
y1 = x 1 6x + 24x2 y y2 = x 2
x3 3x4 + 5 x 5 x + ::: .

**************** Corregir este ejemplo **********************************************************


Ejemplo.
Determinar dos soluciones Linealmente Independientes, por el método de
Frobenius, alrededor del punto singular regular x0 = 0, de la ecuación

xy 00 + 2y 0 + 9xy = 0 (1)

Solución.

Escribimos la ecuación de la siguiente forma:


2 0
y 00 + y + 9y = 0
x

2
Donde: P (x) = y Q(x) = 9. Vemos que x0 = 0; es un punto singular
x
regular de la ecuación (1) siendo
2 2x
p0 = lim (x x0 )P (x) = lim (x x0 ) = lim = lim 2 = 2
x!x0 x!0 x x!0 x x!0

q0 = lim (x x0 )2 Q(x) = lim (x x0 )2 9 = lim 9x2 = 0


x!x0 x!0 x!0

De manera que la ecuación inicial es


r(r 1) + p0 r + q0 = 0

r(r 1) + 2r + 0 = 0

r (r 1 + 2) = 0

r (r + 1) = 0

r1 = 0

r2 + 1 = 0 r2 = 1

Como r1 r2 = 0 ( 1) = 1, la diferencia de las raices es un entero positivo,


entonces una solución linealmente independiente es:

106
1
X 1
X
y = xr an xn = an xn+r
n=0 n=0

Sus derivadas son:


1
X
y0 = (n + r)an xn+r 1

n=0

1
X
y 00 = (n + r 1)(n + r)an xn+r 2
.
n=0

Sutituyendo y y sus derivadas en (1), tenemos


1
X 1
X 1
X
x (n + r 1)(n + r)an xn+r 2
+2 (n + r)an xn+r 1
+ 9x an xn+r = 0
n=0 n=0 n=0

1
X 1
X 1
X
n+r 1 n+r 1
(n + r 1)(n + r)an x +2 (n + r)an x +9 an xn+r+1 = 0
n=0 n=0 n=0

1
X 1
X
(n + r) [(n + r 1 + 2)] an xn+r 1
+9 an xn+r+1 = 0
n=0 n=0

1
X 1
X
n+r 1
(n + r)(n + r + 1)an x +9 an xn+r+1 = 0
n=0 n=0

Haciendo: n n+1 y n n 1, respectivamente, tenemos

1
X 1
X
(n + r + 1)(n + r + 2)an+1 xn+r + 9 an 1x
n+r
=0
n= 1 n=1

Desarrollamos dos términos para la primeria serie, tenemos


Para r = r1 = 0, se tiene

107
1
X
r(r + 1)a0 xr 1
+ (r + 1)(r + 2)a1 xr + (n + r + 1)(n + r + 2)an+1 xn+r +
n=1

1
X
n+r
9 an 1x =0
n=1
1
X
[r(r + 1)a0 xr ] x 1
+ (r + 1)(r + 2)a1 xr + (n + r + 1)(n + r + 2)an+1 xn+r +
n=1

1
X
n+r
9 an 1x =0
n=1
1
X
1
r(r + 1)a0 x xr + (r + 1)(r + 2)a1 xr + (n + r + 1)(n + r + 2)an+1 xn+r +
n=1
1
X
n+r
9 an 1x =0
n=1
1
r(r + 1)a0 x + [r(r + 1)a0 + (r + 1)(r + 2)a1 ] xr +
1
X
n+r
[(n + r + 1)(n + r + 2)an+1 + 9an 1] x =0
n=1

Igualando a cero los coe…cientes de potencias iguales de x, tendremos


r(r + 1)a0 = 0

r(r + 1)a0 + (r + 1)(r + 2)a1 = 0 como r = 0, entonces a1 = 0:

(n + r + 1)(n + r + 2)an+1 + 9an 1 =0

La ecuación inicial es r(r + 1) = 0, entonces r1 = 0 y r2 = 1. Nótese que

esta ecuación indicial es el mismo que se obtuvo más arriba.

Como se veri…ca las raices son las mismas anteriormente encontradas. Siendo
r(r + 1)a0 ra0
a1 = =
(r + 1)(r + 2) r+2

Para n 1; la fórmula de recurrencia general es:


9an 1 32 an 1
an+1 = = ; n = 1; 2; 3; : : :
(n + r + 1)(n + r + 2) (n + r + 1)(n + r + 2)

Para la raíz más pequeña r2 = 1, tenemos

108
32 an 1 32 an 1
an+1 = = ; n = 1; 2; 3; : : :
(n 1 + 1)(n 1 + 2) n(n + 1)

ra0
Siendo a1 = :
r+2

Para r = r2 = 1; tenemos:
1a0 a0
a1 = = = a0
1+2 1

Sí: n = 1; 2; 3; : : :
32 a0 32 32
a2 = = a0 = a0
1(1 + 1) 2 2!

32 a1 32 32
a3 = = a0 = a0
2(2 + 1) 2 3 3!

32 a2 32 32 32 32 34
a4 = = a0 = a0 = a0 =
3(3 + 1) 3 4 2! 3 4 1 2 1 2 3 4

34
a0
4!

Sustituyendo estos coe…cientes en la primera ecuación obtenemos


1
X
r2
y1 = x an xn = x 1
a0 + a1 x + a2 x2 + a3 x3 +
n=0

1 32 32 34
y1 = x a0 + a0 x a0 x2 a0 x3 + a0 x4 +
2! 3! 4!

1 32 2 32 3 34 4
y1 = a0 1 + x x x + x +
x 2! 3! 4!

1 32 32 2 34 3
y1 = a0 +1 x x + x + :
x 2! 3! 4!

La segunda solución Linealmente Independiente podemos obtener mediante


el método de reducción de orden conociendo una solución y1 .
Z R
1 P (x)dx
y2 = y1 e dx
y12

109
2
Siendo P (x) = ; de manera que:
x
Z R 2 Z R 1 Z
1 dx 1 2 dx 1 2 lnjxj
y 2 = y1 e x dx = y 1 e x dx = y 1 e dx
y12 y12 y12
Z Z Z Z
1 lnjxj 2 1 2 1 1 1
y 2 = y1 2 e dx = y 1 2 x dx = y 1 2 2
dx = y 1 2 dx
y1 y1 y1 x y1 x2
Z
1
y 2 = y1 " 2
# dx
2
2
1 3 32 2 34 3
x a0 +1 x x + x +
x 2! 3! 4!
Z
1
y2 = y1 a0 2 dx
1 9 3 2 27 3
x2 +1 x x + x +
x 2 2 8
Z
1
y2 = a0 y1 dx
1 81 9 729 6 2 27 2
x2 + 1 + x2 + x4 + x + + 9 3x + x + 9x 3x2 +
x2 4 4 64 x 4
Z
1
y2 = a0 y1 dx
1 2
x2 + 8 11x + 27x2 +
x2 x
Z
1
y2 = a0 y1 dx
1 + 2x 8x2 11x3 + 27x4 +
Z
1
y2 = a0 y1 1 +2 8x 11x2 + 27x3 + dx

Z Z Z Z Z
1
y2 = a0 y1 1 dx + 2 dx 8 xdx 11 x dx + 27 x3 +
2
dx

11 3 27 4
y2 = a0 y1 x + 2x 4x2 dx x + x + .
3 4

TEMA No 5

LA TRANSFORMADA DE LAPLACE.
DEFINICIÓN
Sea f (t) una función continua en un intervalo [0; 1) y suponemos que f
satisface ciertas condiciones. Entonces la integral

110
Z 1
st
L ff (t)g = F (s) = e f (t) dt
0

se denomina "Transformada de Laplace" de f , siempre y cuando la integral


sea convergente.

Notación.
Sean f (t), g (t) y y (t) funciones continuas, cuyas transformadas de Laplace
son

L ff (t)g = F (s), L fg (t)g = G (s), L fy (t)g = y (s)

CONDICIONES SUFICIENTES
Las condiciones su…cientes que garantizán la existencia de la transformada de
f (t), L ff (t)g, son que f sea continua parte por parte, en [0; 1) y que f sea de
orden exponencial para t > T . La primera condición su…ciente, geometricamente
signi…ca que:

ORDEN EXPONENCIAL

De…nición.
Se dice que una función f , es de orden exponencial sí existen números a > 0,
M > 0 y T > 0 tales que jf (t)j 6 M eat para t > T .
Así por ejemplo, sí f es una función creciente, entonces la condición jf (t)j 6
M eat , t > T , simplemente establece que la grá…ca de f en un intervalo [T; 1)
no crece más rapido que la grá…ca de la función exponencial M eat , donde a es
una constante positiva, gra…camente esto es.

111
OrdenExp

1 4:pdf

Ejemplo.
t
Las funciones f (t) = t, f (t) = e y f (t) = 2 cos t son de orden exponencial
para t > 0, puesto que se veri…ca.

jtj 6 et , je t j 6 et y j2 cos tj 6 2et

Gra…camente esto signi…ca

Orden Lineal Orden Exp Neg

1 5:pdf 1 6:pdf

2
Una función tal como f (t) =et no es del orden exponencial puesto que su
grá…ca crece más rapido que cualquier función exponencial M eat para a > 0,
gra…camente esto es
N oOrdenExp

1 8:pdf

112
L Es una Transformación Lineal
Teorema.
Supongase que f (t) y g (t) son dos funciones continuas, cuyas transformadas
de Laplace existen, para s > , s > , entonces

Z 1
L [ f (t) + g (t)] = e st [ f (t) + g (t)] dt =
Z 1 Z 1 0

e st f (t) dt+ e st g (t) dt = L ff (t)g+ L fg (t)g = F (s)+ G (s) :


0 0

Debido a esta propiedad dada, se dice que L es una transformación lineal,


siempre que ambas integrales sean convergentes

Ejemplo 1.

Sea:f (t) = 1 con, t > 0; hallar L ff (t)g por de…nición:

Solución.
Z 1 Z b st bs
st st e e 1
L f1g = e (1) dt = lim e dt = lim jb0 = lim + =
0 b!1 0 b!1 s b!1 s s

bs
e 1 1
lim + lim = 0 + .
b!1 s b!1 s s
1
De manera que F (s) = , s > 0
s

Ejemplo 2.

Sea f (t) = t con, t > 0,hallar L ff (t)g por de…nición.


Solución. Z 1
st
Donde L ff (t)g = e tdt
0

Integrando por partes, tenemos

Sí: u = t ) du = Zdt
st
st st e
dv = e )v= e dt =
s
Z 1 st Z 1 st st Z 1
st e e e 1
L ff (t)g = e tdt = t j1
0 + dt = t j1
0 + e st
dt =
0 s 0 s s s 0

1 e st 1 st
0+0 j , Sí: e = 0, cuando t ! 1, entonces
s s 0

113
1 1
: = 0+
s s
1
F (s) = 2 , s > 0 .
s
Ejemplo 3.

Sea: f (t) = t2 con , t > 0;hallar L ff (t)g por de…nición:


Solución. Z 1
2
Donde L t = e st t2 dt
0

Integrando por partes

Sí: u = t2 ) du =Z 2t dt
st
st st e
dv = e )v= e dt =
s
Z 1 st Z
e 2 2
L t2 = e st 2
t dt = t2 j1
0 + e st
tdt = 0+0+ L ftg =
0 s s s

2 1 2
= ; s > 0.
s s2 s3

En general
n!
L ftn g = , para todo n = 1; 2; 3; : : :
sn+1
Ejemplo 4.

Sea: f (t) = e 2t
con , t > 0;hallar L ff (t)g por de…nición:
Solución.
Z 1 Z 1
2t st 2t (s+2)t e (s+2)t 1
Donde L e = e e dt = e dt = j = 0+
0 0 (s + 2) 0
1
.
s+2
1
F (s) = , s> 2.
s+2

Ejemplo 5.

Sea: f (t) = sen at con, t > 0: Hallar L ff (t)g por de…nición:

Solución. Z 1
st
Donde L fsen atg = e sen at dt
0

114
Integrando por partes, tenemos

st st
Sí: u = e ) du =
Z se dt
cos at
dv = sen at ) v = sen at dt = .
a

Luego, Z Z
1 1
st st cos at 1 s st
L fsen atg = e sen at dt = e j0 e cos at dt .
0 a a 0

Integrando nuevamente por partes, tenemos

st st
Sí: u = e ) du Z
= se dt
sen at
dv = cos at ) v = cos atdt = .
a

De donde Z 1 Z 1
st 1 s sen at 1 s
L fsen atg = e sen at dt = 0+ e st j + e st
sen at dt
0 a a a 0 a 0
Z 1
1 s s
= 0 0+ e st sen at dt
a a a 0
1 s2
L fsen atg = L fsen atg
a a2

s2 1
L fsen atg + L fsen atg =
a2 a

s2 1
1+ L fsen atg =
a2 a

1
L fsen atg = .
a + s2
2
a
a2

De manera que
a
F (s) = 2 ,s>a.
s + a2
Ejemplos.
Utilizando la tabla y la linealidad de la transformada de Laplace, determinar
la transformada de la siguientes funciones.

1). Sea: f (t) = 3t3 2t2 + t 5


L ff (t)g = L 3t3 2t2 + t 5 = 3L t3 2L t2 + L ftg 5L f1g =

115
3! 2! 1 1
3 2 +1 5
s4 s3 s2 s
18 4 1 5
F (s) = + 2 , s>0.
s4 s3 s s

3
2). Sea: f (t) =n(t 1) o
3
L ff (t)g = L (t 1) = L t3 3t2 + 3t 1 = L t3 3L t2 +

3! 2! 1 1
3L ftg L f1g = 3 +3
s4 s3 s2 s
6 6 3 1
F (s) = 4 3
+ 2 , s>0.
s s s s

3). Sea: f (t) = 4e 5t + 3 sen t 2 cos 4t


L ff (t)g = L 4e 5t + 3 sen t 2 cos 4t = 4L e 5t
+3L fsen tg 2L fcos 4tg

1 1 s
=4 +3 2
s 5 s2 + 12 s2 + 42

4 3 2s
F (s) = + , s> 5.
s 5 s2 + 12 s2 + 42

4). Sea: f (t) = e 2t t4 + 2t2 1


L ff (t)g = L e 2t t4 + 2t2 1 = L e 2t 4
t + 2e 2t 2
t e 2t

" #
4! 2!
= L t4 e 2t
+ 2L t2 e 2t
L e 2t
= 5 +2 3
(s + 2) (s + 2)

1
s+2
24 4 1
F (s) = 5 + 3 , s> 2.
(s + 2) (s + 2) s+2

5). Sea: f (t) = e2t sen 3t


3 3
L ff (t)g = L e2t sen 3t = 2 =
(s 2) + 32 s2 4s + 4 + 9

3
F (s) = .
s2 4s + 13

6t
p 1 p
6). Sea: f (t) = e cos 2t t sen 2 3t + 5
2

116
6t
p 1 p
L ff (t)g = L e cos 2t t sen 2 3t + 5
2

6t
p 1 p
=L e cos 2t L t sen 2 3t + 5L f1g
2 0 1
p
s+6 1B 4 3s C 1
= p 2 @h p 2 i2 A + 5
(s + 6) +
2
2 2 2 s
s + 2 3
p
s+6 2 3s 5
F (s) = 2 + s, s > 0 .
s2 + 12s + 38 2
(s + 12)

7). Sea: f (t) = sen 4t cos 4t


1 1 1 8
L ff (t)g = L fsen 4t cos 4tg = L f2 sen 4t cos 4tg = L fsen 8tg =
2 2 2 s2 + 82

4
F (s) = .
s2 + 64

8). Sea: f (t) = cos 2t cos t


L ff (t)g = L fcos 2t cos tg = L cos2 t sen2 t cos t = L cos3 t sen2 t cos t

cos 3t + 3 cos t 1 cos 2t


= L cos3 t L sen2 t cos t = L L cos t
4 2
1 3 1 1
= L fcos 3tg + L fcos tg L fcos tg + L fcos 2t cos tg
4 4 2 2
Luego, escribimos
1 3 1
L fcos 2t cos tg 21 L fcos 2t cos tg = L fcos 3tg + L fcos tg L fcos tg
4 4 2

1 1 3 1
2 L fcos 2t cos tg =
4
L fcos 3tg + L fcos tg
4 2
L fcos tg

1 3 1
= L fcos 3tg + L fcos tg L fcos tg
4 4 2
1 s 3 s 1 s
= +
4 s2 + 32 4 s2 + 12 2 s2 + 12
s 1 s
= + .
4 (s2 + 9) 4 s2 + 12
Por tanto,
s s
L fcos 2t cos tg = + .
2 (s2 + 9) 2 (s2 + 1)

TRANSFORMADA INVERSA DE LAPLACE

117
Ahora estudiamos el proceso inverso de la sección anterior , es decir dada
una función F (s) hallar la función f (t), que corresponde a su transformada. Se
dice que f (t) es la transformada inversa de F (s), entonces escribimos:

1
f (t) = L fF (s)g ,

1
L es una Transformación Lineal.

La transformada inversa de laplace es en si misma una transformación lineal,


esto es, para las constantes y , entonces:
1 1 1
L f F (s) + G (s)g = L fF (s)g + L fG (s)g

donde, F y G son las transformadas de algunas funciones f y g .

Ejemplos.

Dada las siguientes funciones, determinar su transformada inversa.

4
1). Sea: F (s) =
s2 + 9
4 4 3
L 1 fF (s)g = L 1 2
= L 1
.
s +9 3 s2 + 32

Entonces:
4
f (t) = sen 3t
3
4s
2). Sea: F (s) =
4s2 +1 8
9
4s > > ( )
4s <
= s
L 1
fF (s)g = L 1
=L 1 4 =L 1
.
> 2 1 2
: 4s + 1 >
2
4s + 1 ; s2 + 2
4
Entonces:
1
f (t) = cos t .
2
2s 6
3). Sea: F (s) =
s2 + 9
2s 6 s 3
L 1 fF (s)g = L 1 = 2L 1
2L 1
.
s2 + 9 s2 + 32 s2 + 32

118
Entonces:

f (t) = 2 cos 3t 2 sen 3t = 2(cos 3t sen 3t) .

s+1
4). Sea F (s) =
s2 + 2s + 10
s+1 s+1
L 1 fF (s)g = L 1 2
= L 1
=
s + 2s + 10 (s2 + 2s + 12 ) + 10 12
( )
1 s+1
L 2 .
(s + 1) + 32

Entonces:
f (t) =e t cos 3t

1
5). Sea: F (s) =
s2 +s 20 8 9
>
> >
>
1 < 1 =
1 1 1
L fF (s)g = L =L =
s2 +s 20 >
> 1 1 >
>
: s2 + s + 20 + ;
8 9 4 4
>
> 9 >
>
>
< >
=
2 1 2
L 2 2> .
9 >
> 1 9 >
>
: s+ >
;
2 2

Entonces:
2 1
f (t) = e 2t sen h 92 t .
9
3s 15
6). Sea: F (s) =
2s2 4s + 10
1 1 3s 15
L fF (s)g = L 2
2s 4s + 10

Factorizando tenemos:
3 s 5 3 s 5
L 1 fF (s)g = L 1 2
= L 1
=
2 s 2s + 5 2 (s22s + 12 ) + 5 12
3 1 (s 1) 4 3 1 (s 1) 4
L = L =
2 (s 1)2 + 22 2 (s 1)2 + 22

3 1 (s 1) 3 1 2
L 2L
2 (s 1)2 + 22 2 (s 1)2 + 22
3 1 (s 1) 1 2
= L 3L .
2 (s 1)2 + 22 (s 1)2 + 22

119
Entonces:
3 t
f (t) = e cos 2t 3et sen 2t .
2
2s + 16
7). Sea: G(S) =
s2 + 4s + 13
1 1 2s + 16 1 2s + 16
L fG(s)g = L =L
s2 + 4s + 13 (s2 + 4s + 22 ) + 13 22

1 s+8 1 (s + 2) + 6
= 2L = 2L
(s2 + 4s + 22 ) + 9 (s + 2)2 + 32

1 s+2 1 3
= 2L + 4L
(s + 2)2 + 32 (s + 2)2 + 32
1 s+2 1 3
= 2L + 4L .
(s + 2)2 + 32 (s + 2)2 + 32

Entonces:
2t 2t
g(t) = 2 e cos 3t + 2e sen 3t .

8s2
8). Sea: G(s) =
s4 + 8s2 + 16
1 1 8s2 1 8s2
L fG(s)g = L =L
s + 8s2 + 16
4 (s2 + 22 )2
2 2 2s2 (2 2s2 )
= L 1 = 2L 1
. Por
(s2 + 22 )2 (s2 + 22 )2

1 2ks2
L fsen kt + kt cos ktg = .
(s2 + k 2 )2

Entonces:

g(t) = 2 (sen 2t + 2t cos 2t) .

FRACCIONES PARCIALES

Los fracciones parciales, son muy importantes para determinar la transfor-


mada inversa de Laplace. Así consideramos los ejemplos siguientes

Ejemplos
Determinar la transformada inversa de la función:
s2 26s 47
F (s) = .
(s 1)(s + 2)(s + 5)

120
Solución.

Aplicamos fracciones parciales a la función f (s).

s2 26s 47 A B C
= + +
(s 1)(s + 2)(s + 5) s 1 s+2 s+5
A(s + 2)(s + 5) + B(s 1)(s + 5) + C(s 1)(s + 2)
=
(s 1)(s + 2)(s + 5)
s2 26s 47 = A(s + 2)(s + 5) + B(s 1)(s + 5) + C(s 1)(s + 2)

Evaluando

Sí: s = 1, tenemos
(1)2 26(1) 47 = A(3)(6)

18A = 72
72
A= 18

A= 4

Sí:s = 2, tenemos
( 2)2 26( 2) 47 = B( 3)(3)

9B = 9

B= 1.

Sí: s = 5, tenemos
( 5)2 26( 5) 47 = C( 6)( 3)

18C = 108

C=6

Siendo
s2 26s 47 4 1 6
= + + :
(s 1)(s + 2)(s + 5) s 1 s+2 s+5

Luego
1 s2 26s 47 1 4 1 1 1 6
L =L +L +L
(s 1)(s + 2)(s + 5) s 1 s+2 s+5

121
1 1 1 1 1 1
= 4L L +6L
s 1 s+2 s+5

Entonces
f (t) = 4et e 2t
+ 6e 5t
.

Ejemplo
1
Determinar L fF (s)g, donde

s2 + 8
F (s) =
s4 4s2

Solución.

Primero factorizamos el denominador de F (s)

s2 + 8
F (s) =
s4 4s2

Aplicando fracciones parciales a F (s) , tenemos:


s2 + 8 A B C D
2
= + 2+ +
s (s + 2)(s 2) s s s+2 s 2

As(s + 2)(s 2) + B(s + 2)(s 2) + Cs2 (s 2) + Ds2 (s + 2)


=
s2 (s + 2)(s 2)
s2 + 8 = As(s + 2)(s 2) + B(s + 2)(s 2) + Cs2 (s 2) + Ds2 (s + 2)

Hacemos la evaluación

Sí: s = 0, tenemos
8 = B(2)( 2)

B= 2

Sí: s = 2, entonces
( 2)2 + 8 = C( 2)2 ( 4)

16C = 12
3
C=
4

Sí: s = 2, tenemos

122
22 + 8 = D(4)(4)

16D = 12
3
D=
4

Ahora efectuamos la siguiente operación


s2 + 8 = As3 4As Bs2 4B Cs3 2Cs2 + Ds3 + 2Ds2

= (A + B + D)s3 + (B C + 2D)s2 4As 4B .

Igualando los coe…cientes de potencias iguales de s, obtenemos el siguiente


sistema
8
>
> A+C +D =0
<
B 2C + 2D = 1
=> A = 0 B = 2:
>
> 4A = 0
:
4B = 8

Siendo entonces
3 3
s2 + 8 0 2 4 4
= + + +
s2 (s + 2)(s 2) s s2 s+2 s 2

Luego
1 s2 + 8 1 1 3 1 1 3 1 1
L = 2 L L + L
s2 (s + 2)(s 2) s2 4 s+2 4 s 2

Entonces
3
f (t) = 2t % 2t
+ 43 %2t .
4
Ejemplo
1
Determinar L fF (s)g, donde

2s3 + s2 + 3
F (s) =
s4 1
Solución.

Factorizar el denominador de F (s):


2s3 + s2 + 3
F (s) = L 1
(s + 1)(s 1)(s2 + 1)

Aplicando fracciones parciales a F (s), se tiene

123
2s3 + s2 + 3 A B Cs + D
= + + 2
(s + 1)(s 1)(s2 + 1) (s + 1) s 1 s +1

A(s 1)(s2 + 1) + B(s + 1)(s2 + 1) + (s + 1)(s 1)(Cs + D)


=
(s + 1)(s 1)(s2 + 1)
2s3 + s2 + 3 = A(s 1)(s2 + 1) + B(s + 1)(s2 + 1) + (s + 1)(s 1)(Cs + D)

2s3 +s2 +3 = As3 +As As2 A+Bs3 +Bs+Bs2 +B +Cs3 Cs+Ds2 D

= (A+B +C)s3 +( A+B +D)s2 +(A+B C)s+( A+B D)

Igualando los coe…cientes


8 de potencias iguales de s ,Obtenemos el sistema
>
> A+B+C =2 (1)
<
A+B+D =1 (2)
>
> A +B C =0 (3)
:
A+B D =3 (4)

Resolviendo el sistema obtenemos que: A = 21 ; B = 32 ; C = 1; D = 1:


Luego
1 3
2s3 + s2 + 3 2 2 s 1
= + + :
(s + 1)(s 1)(s2 + 1) s + 1 s 1 s2 + 1

Siendo
1 2s3 + s2 + 3 1 1 1 3 1 1 1 s
L = L + L +L
(s + 1)(s 1)(s2 + 1) 2 s+1 2 s 1 s2 + 1

1 1
L :
s2 + 1
Entonces
1
f (t) = 2e
t
+ 32 et + cos t sen t .

LA TRANSFORMADA DE LAPLACE DE LA DERIVADA

Teorema A.
Sea f (x) una función continua en [0; 1) y que f 0 (x) es continua parte por
parte en [0; 1) siendo ambos de orden exponencial a. Entonces, para s > a

L ff 0 (t)g = sL ff (t)g f (0)

Demostración.
Para demostrar este Teorema, suponemos que existe L ff 0 (t)g, siendo

124
Z 1
L ff 0 (t)g = e st
f 0 (t) dt.
0

Integrando por partes, tenemos

Sí: u =e st
) du = Z se st dt
dv = f 0 (t) ) v = f 0 (t) dt = f (t).

Prosiguiendo Z Z
1 1
L ff 0 (t)g = e f (t) dt =e
st 0 st
f (t) j1
0 + s e st
f (t) dt = f (0) +
0 0

sL ff (t)g.

En consecuencia

L ff 0 (t)g = sL ff (t)g f (0).

ó bien

L ff 0 (t)g = sF (s) f (0).

donde

F (s) = L ff (t)g :

Nótese que, e st f (t) = 0 cuando t ! 1.


En forma análoga podemos obtener L ff 00 (t)g. Vemos que la transformada
es de la siguiente manera
Z 1
L ff 00 (t)g = e st f 00 (t) dt.
0

125
Integrando por partes

Sí: u =e st ) du = se st dt
dv = f 00 (t) dt ) v = f 0 (t).

Prosiguiendo Z Z
1 1
L ff 00 (t)g = e f (t) dt =e
st 00 st
f 0
(t) j1
0 + e st
f 0
(t) dt = f
0 0

0
(0) + s [sL ff (t)g f (0)]

En consecuencia
00
L ff (t)g = s2 L ff (t)g sf (0) f 0 (0).

En general, resulta
(n)
L f (t) = sn L ff (t)g sn 1
f (0) sn 2
f 0 (0) f (n 1)
(0)

donde
F (s) = L ff (t)g :

Ejemplo.

Determinar L fcos atg, utilizando el Teorema A..


Solución.

Sea: f (t) = cos at


0
Donde f (0) = cos a0 = 1 y f (t) = a sen at, luego sustituimos en el
Teorema A.
L f a sen atg = sL fcos atg 1

aL fsen atg = sL fcos atg 1


a
a 2 = sL fcos atg 1
s + a2
a2
sL fcos atg = 1
s2 + a2

a2
1 s2 + a2 a2
L fcos atg = =
s
+a )2s (s2 s (s2 + a2 )
2
s s
L fcos atg = = 2 .
s (s2 + a2 ) s + a2

126
APLICACIONES
Resolución de Problemas de Valor Inicial
La transformada de Laplace, se puede aplicar para resolver problemas de
valor inicial a ecuaciones diferenciales lineales con coe…cientes constantes.

Ejemplo.
Utilizando la transformada de Laplace, resolver el problema de valor inicial

y 0 + 3y =e 3t
, y (0) = 4

Solución.
Aplicando la transformada de Laplace a ambos miembros de la ecuación,
tenemos

L fy 0 + 3yg = L e 3t

Aplicando linealidad de la transformada, se tiene

L fy 0 g + 3L fyg = L e 3t

sL fyg y (0) + 3L fyg = L e 3t


1
sY (s) 4 + 3Y (s) =
s+3
1
Y (s) (s + 3) 4 =
s+3
1
Y (s) (s + 3) = +4
s+3
1 4
Y (s) = 2 + s + 3.
(s + 3)

Ahora, calculamos la transformada inversa de Y (s), y esto es


( )
1 4
L 1 fY (s)g = L 1 2 + s+3
(s + 3)
( )
1 1 1 1
L fY (s)g = L 2 + 4L 1 .
(s + 3) s+3

Por tanto, la solución es


3t 3t
y (t) = te + 4e .

127
Ejemplo.
Utilizando la transformada de Laplace , resolver el problema de valor inicial.

y 00 4y 0 + 4y = t3 e2t ; y (0) = 0 , y 0 (0) = 0

Solución.

Aplicamos L y linealidad

L fy 00 g 4L fy 0 g + 4L fyg = L t3 e2t

s2 L fyg sy (0) y 0 (0) 4 (sL fyg y (0)) + 4L fyg = L t3 e2t


6
s2 Y (s) 4sY (s) + 4Y (s) = 4
(s 2)

6
Y (s) s2 4s + 4 = 4
(s 2)

2 6
Y (s) (s 2) = 4
(s 2)
6
Y (s) = 6.
(s 2)

1
Ahora, calculamos L y la linealidad,
( )
1 1 6
L fY (s)g = L 6
(s 2)
( )
1 1 1
L fY (s)g = 6L 6
(s 2)
( )
6 5!
L 1 fY (s)g = L 1
6
5! (s 2)

1 5 2t
y (t) = t e .
20

Ejemplo.

Utilizando la transformada de Laplace, resolver el problema de valor inicial

128
y 00 + 2y 0 + 5y = 3e t
sen t; y (0) = 0 , y 0 (0) = 3.

Solución.

Aplicando L y linealidad, tenemos


L fy 00 g + 2L fy 0 g + 5L fyg = 3L fe t
sen tg

s2 L fyg sy (0) y 0 (0) + 2 (sL fygy (0)) + 5L fyg = 3L fe t


sen tg
" #
1
s2 Y (s) 3 + 2sY (s) + 5Y (s) = 3 2
(s + 1) + 12

3
s2 Y (s) + 2sY (s) + 5Y (s) = +3
s2 + 2s + 2
3
Y (s) s2 + 2s + 5 = +3
s2 + 2s + 2
3 3
Y (s) = + .
(s2 + 2s + 5) (s2 + 2s + 2) s2 + 2s + 5

Aplicando fracciones parciales, tenemos


3 As + B Cs + D (As + B) s2 + 2s + 2 + (Cs + D) s2 + 2s + 5
2 2
= 2 + 2 =
(s + 2s + 5) (s + 2s + 2) s + 2s + 5 s + 2s + 2 (s2 + 2s + 5) (s2 + 2s + 2)

As3 + 2As2 + 2As + Bs2 + 2Bs + 2B + Cs3 + 2Cs2 + 5Cs + Ds2 + 2Ds + 5
=
(s2 + 2s + 5) (s2 + 2s + 2)

(A + C) s3 + (2A + B + 2C + D) s2 + (2A + 2B + 5C + 2D) s + (2B + 5D)


= :
(s2 + 2s + 5) (s2 + 2s + 2)

3 (A + C) s3 + (2A + B + 2C + D) s2 + (2A + 2B + 5C + 2D) s + (2B + 5D)


= .
(s2 2
+ 2s + 5) (s + 2s + 2) (s2 + 2s + 5) (s2 + 2s + 2)
Igualando los numeradores, se tiene
0s3 +0s2 +0s+3 = (A + C) s3 +(2A + B + 2C + D) s2 +(2A + 2B + 5C + 2D) s+

(2B + 5D).

Igualando, los coe…cientes de potencias iguales de s del primer y segundo


miembro,
8 obtenemos el sistema
>
> A + C=0 (1)
<
2A + B + 2C + D = 0 (2)
>
> 2A + 2B + 5C + 2D = 0 (3)
:
2B + 5D = 3 (4)

De la primera ecuación despejamos C = A y reemplasando en la segunada


ecuación del sistema, obtenemos

129
0.1.1 2A + B 2A + D = 0 ) B + D = 0.

Luego, resolvemos el sistema


B+D =0 j 5
2B + 5D = 3 j
——————–
5B 5D = 0
2B + 5D = 3
——————–
3B =3 ) B= 1. Demanera que: D = 1:

Luego, reemplazando B = 1 y D = 1 en la tercera ecuación del sitema, se


obtiene
2A + 5C = 0.
Sustituyendo C = A, en esta ecuación, se obtiene
2A + 5 ( A) = 0 ) A=0 y C = 0.

En consecuencia, las raices del sistema son: A = 0, B = 1, C = 0 y


D = 1.
Siendo
3 1 1
= 2 + 2
(s2 2
+ 2s + 5) (s + 2s + 2) s + 2s + 5 s + 2s + 2
Continuando
1 1 3 2 1
Y (s) = 2 + 2 + 2 = 2 + 2
s + 2s + 5 s + 2s + 2 s + 2s + 5 s + 2s + 5 s + 2s + 2

De manera que
1 1 1 1 1 1 3
L fY (s)g = L +L +L
s2 + 2s + 5 s2 + 2s + 2 s2 + 2s + 5

1 2 1 1
=L +L
s2 + 2s + 5 s2 + 2s + 2

1 2 1 1
=L +L
s2 + 2s + 5 s2 + 2s + 2

1 2 1 1
=L +L
(s2 + 2s + 12 ) + 5 12 (s2 + 2s + 12 ) + 2 12
( ) ( )
1 2 1 1
=L 2 +L 2
(s + 1) + 4 (s + 1) + 1

130
( ) ( )
1 1 2 1 1
L fY (s)g = L 2 +L 2 .
(s + 1) + 4 (s + 1) + 1

Finalmente:
t t
y (t) = e sen 2t + e sen t .

Ejemplo.
Mediante la transformada de Laplace, resolver el problema de valor inicial.

y 00 y0 2y = 2 sen t 8 cos t ; y = 1 , y0 = 0.
2 2

Solución.
Donde

L fy 00 g L fy 0 g 2L fyg = 2L fsen tg 8L fcos tg


h i
s2 L fyg sy y0 sL fyg y 2L fyg =
2 2 2
2L fsen tg 8L fcos tg
2 1 s
s Y (s) s [sY (s) 1] 2Y (s) = 2 2 8
s + 12 s2 + 12
1 s
s2 Y (s) s sY (s) + 1 2Y (s) = 2 2 8
s + 12 s2 + 12

2 8s
s2 s 2 Y (s) + 2s+1=
+1 2 s + 12 s2
2 8s
(s 2) (s + 1) Y (s) = 2 +s 1
s +1
2 8s s 1
Y (s) = 2
+
(s 2) (s + 1) (s + 1) (s 2) (s + 1)
8s 2 + (s 1) s2 + 1 8s 2 + s3 + s s2 1
Y (s) = =
(s 2) (s + 1) (s2 + 1) (s 2) (s + 1) (s2 + 1)
s3 s2 7s 3
Y (s) = .
(s 2) (s + 1) (s2 + 1)

Aplicando fraccciones parciales, tenemos


s3 s2 7s 3 A B Cs + D
= + + 2
(s 2) (s + 1) (s2 + 1) s 2 s+1 s +1
A (s + 1) s2 + 1 + B (s 2) s2 + 1 + (Cs + D) (s 2) (s + 1)
=
(s 2) (s + 1) (s2 + 1)

(As + A) s2 + 1 + (Bs 2B) s2 + 1 + Cs2 + D s2 s 2


=
(s 2) (s + 1) (s2 + 1)

131
As3 + As + As2 + A + Bs3 + Bs 2Bs2 2B + Cs3 Cs2 2Cs + Ds2 Ds
=
(s 2) (s + 1) (s2 + 1)

As3 + Bs3 + Cs3 + As2 2Bs2 Cs2 + Ds2 + As + Bs 2Cs Ds + A 2B


=
(s 2) (s + 1) (s2 + 1)

(A + B + C) s3 + (A 2B C + D) s2 + (A + B 2C D) s + (A 2B 2D)
= .
(s 2) (s + 1) (s2 + 1)

o bien
s3 s2 7s 3 (A + B + C) s3 + (A 2B C + D) s2 + (A + B 2C D) s + (A 2B 2D)
2
= .
(s 2) (s + 1) (s + 1) (s 2) (s + 1) (s2 + 1)
Simpli…cando los denominadores, se tiene
s3 s2 7s 3 = (A + B + C) s3 +(A 2B C + D) s2 +(A + B 2C D) s+
(A 2B 2D) (1)

Evaluación:

Sí: s = 2 en (1), entonces


23 22 7 (2) 3 = (A + B + C) 23 +(A 2B C + D) 22 +(A + B 2C D) 2+

(A 2B 2D)
8 4 14 3 = 8A + 8B + 8C + 4A 8B 4C + 4D + 2A + 2B 4C

2D + A 2B 2D
13
13 = 15A )) A = .
15

Sí: s = 1 en (1), entonces


1 1+7 3 = (A + B + C) ( 1)+(A 2B C + D)+(A + B 2C D) ( 1)+

(A 2B 2D)
2= A B C +A 2B C +D A B + 2C + D + A 2B 2D

1
2= 6B )) B = .
3

Luego, igualando los coe…cientes de potencias iguales de s en (1), obtenemos


el sistema
8
>
> A+B+C =1 (1)
<
A 2B C + D = 1 (2)
>
> A + B 2C D = 7 (3)
:
A 2B 2D = 3 (4)

132
13 1
Como ya conocemos de antemano las raíces A = y B = , susti-
15 3
33
tuyendo estos valores en el sistema obtenemos los demás raices, esto es: C =
15
7
yD= .
5
Operaciones Auxiliares:
Sea la ecuación
A+B+C =1
13 1
+C =1
15 3
13 1 33
C =1+ + )) C = .
15 3 15
Reemplazamos en la segunda ecuación los valores de A, B ; C y D, tenemos
A 2B C + D = 1
13 1 33
2 +D = 1
15 3 15
13 2 33
+ +D = 1
15 3 15
13 + 10 33
+D = 1
15
10 46
+D = 1
15
36
+D =1
15
36
D= 1
15
36 15 21 7
D= = )) D = .
15 15 5
Continuamos, reemplazando los valores de A, B, C y D en la función Y (s):

13 1 33 7
s3 s2 7s 3 s+
Y (s) = = 15 + 3 15
+ 2 5
(s 2) (s + 1) (s2 + 1) s 2 s+1 s +1
13 1 1 1 33 s 7 1
Y (s) = + + .
15 s 2 3 s+1 15 s2 + 1 5 s2 + 1

Ahora calculamos la transformada inversa de Y (s)


1
L fY (s)g =

13 1 1 1 1 1 33 1 s 7 1 1
L L + L + L .
15 s 2 3 s+1 15 s2 +1 5 s2 +1

En consecuencia, la solución es:


13 2t 1 33
y (t) =
15
e 3
e t
+
15
cos t+

133
7
sen t .
5
Ejemplo.

Mediante la transformada de Laplace, resolver el problema de valor incial.

y 00 + y 0 2y = cos 2t ; y (0) = 1 , y 0 (0) = 4.

Solución.
L fy 00 g + L fy 0 g 2L fyg = L fcos 2tg

s2 L fyg sy (0) y 0 (0) + sL fyg y (0) 2L fyg = L cos2 t L sen2 t

s2 + 2 2
s2 Y (s) s 4 + sY (s) 1 2Y (s) =
s (s2 + 4) s (s2 + 4)

s2 + 2 2
s2 + s 2 Y (s) s 5=
s (s2 + 4) s (s2 + 4)

s2
(s + 2) (s 1) Y (s) = +s+5
s (s2 + 4)
s2 s+5
Y (s) = 2
+
s (s + 4) (s + 2) (s 1) (s + 2) (s 1)

Simpli…cando s en la primera fracción y despejando Y (s), se tiene


s s+5
Y (s) = 2 +
(s + 4) (s + 2) (s 1) (s + 2) (s 1)
s + s2 + 4 (s + 5) s + s3 + 5s2 + 4s + 20
Y (s) = 2 = 2 .
(s + 4) (s + 2) (s 1) (s + 4) (s + 2) (s 1)
Simpli…cando los términos semejantes del numerador, resulta
s3 + 5s2 + 5s + 20
Y (s) = 2 .
(s + 4) (s + 2) (s 1)

Aplicando fracciones parciales, tenemos


s3 + 5s2 + 5s + 20 As + B C D
2
= 2 + + .
(s + 4) (s + 2) (s 1) s +4 s+2 s 1

Operamos en el segundo miembro

As + B C D (As + B) (s + 2) (s 1) + C s2 + 4 (s 1) + D s2 + 4 (s + 2)
+ + =
s2 + 4 s + 2 s 1 (s2 + 4) (s + 2) (s 1)
(As + B) s + s 2 + C s3 s2 + 4s 4 + D s3 + 2s2 + 4s + 8
2
=
(s2 + 4) (s + 2) (s 1)

134
As3 + As2 2As + Bs2 + Bs 2B + Cs3 Cs2 + 4Cs 4C + Ds3 + 2Ds2 + 4D
=
(s2 + 4) (s + 2) (s 1)

As3 + Cs3 + Ds3 + As2 + Bs2 Cs2 + 2Ds2 2As + Bs + 4Cs + 4Ds 2B 4
=
(s2 + 4) (s + 2) (s 1)

(A + C + D)s3 + (A + B C + 2D)s2 + ( 2A + B + 4C + 4D)s + ( 2B 4C +


=
(s2 + 4) (s + 2) (s 1)

O bien
s3 + 5s2 + 5s + 20 (A + C + D)s3 + (A + B C + 2D)s2 + ( 2A + B + 4C + 4D)s + ( 2B 4C
=
(s2 + 4) (s + 2) (s 1) (s2 + 4) (s + 2) (s 1)

Simpli…camos los denominadores

s3 + 5s2 + 5s + 20 = (A + C + D)s3 + (A + B C + 2D)s2 + ( 2A + B +


4C + 4D)s + ( 2B 4C + 8D)
Igualando los coe…cientes de potencias iguales s del primer y segundo miem-
bro,8tenemos el siguiente sistema
>
> A+C +D =1 (1)
<
A + B C + 2D = 5 (2)
>
> 2A + B + 4C + 4D = 5 (3)
:
2B 4C + 8D = 20 (4)

3 1
Resolviendo el sistema encontramos que la raíces son: A = , B = ,
20 10
11 31
C= ,D= .
12 15
Continuamos, reemplazando los valores de A, B, C y D en la función Y (s):
3 1 11 31
20 s + 10 12 15
Y (s) = 2
+
s +4 s+2 s 1
3 1 1 1 11 1 31 1
Y (s) = + + .
20 s2 + 4 10 s2 + 4 12 s+2 15 s 1

Ahora calculamos la transformada inversa de Y (s)


3 s 1 1 11 1
L 1 fy (s)g = L 1 + L 1 L 1
+
20 s2 + 4 10 s2 + 4 12 s+2

31 1 1
L .
15 s 1

En consecuencia, la solución es:

135
3 11 31 t
y (t) =
20
cos 2t + 1
20 sin 2t
12
e 2t
+
15
e.

136

S-ar putea să vă placă și